Download as docx, pdf, or txt
Download as docx, pdf, or txt
You are on page 1of 69

LEARNING MODULE

THE NOTRE DAME OF MASIAG, INC.


SY 2020-2021

Statistics and Probability


Grade 11
Quarter 3

Name ______________________________________________

Section _________________ST. JOSEPH___________________

Subject Teacher ________________LANY T. CATAMIN_______________

Class Adviser ___________DANTE H. LAGNAODA, JR._____________

FOR PRIVATE USE


in the Archdiocesan Notre Dame Schools of Cotabato
Strictly not for Public Circulation
QUARTER 3
CHAPTER I: RANDOM VARIABLES AND PROBABILITY DISTRIBUTION
Introductory Part

Overview & References


In the study of basic probability, you have learned that an experiment is any activity which can be
done repeatedly under similar condition. The set of all possible outcomes of an experiment is called the
sample space. You have also learned how to mathematically list the possible outcome of a given experiment.
In tossing a coin, for instance, the possible outcomes are turning up a head or a tail.
Here are the references that you will find useful for this Module:

Albay, Eduard M.,Statistics and Probability, 2nd ed.Makati City: DIWA Textbooks, 2019
Belecina, Rene R, Elisa S. Baccay, and Efren B. Mateo. Statistics and Probability.
Sampaloc, Manila, Rex Bookstore, 2016.
Definition of Probability Distribution. Retrieved from: http://statisticsbyjim.com/basics/-probability-
duistributions/.Date Retrieved: January 24, 2019
Definition of Terms
Random Variable - is a numerical quantity that t- distribution - is also known as Student’s t-
is derived from the outcomes of random distribution, is a probability distribution which is
experiments. utilized in estimating parameters of a certain
A histogram - is a bar graph. population in case of the sample size is small
Variance of a Discrete Random Variable X- and/or the population variance or standard
denoted by σ x , is the square root of the Variance deviation is unknown.
Normal Probability Distribution - is a Degrees of freedom - refer to the number of
probability distribution of a continuous random independent observations in a given set of data.
variables. Percentile - is one of the measurements in
Probability notations - are commonly used to statistics which tells the value below in which an
express a lengthy idea into symbols concerning observations’ percentage in a set of observations
the normal curve. falls
Population - is the group you want to
generalize. It consists of all the members of the
group you are interested in. Sample - is the
subset from the population you want to examine.
Sampling is a process used in statistical analysis
in which a predetermined number of observations
are taken from a larger population
Parameter – is a measure that describes a
population.
Statistic A measure that describes a sample.
A sampling distribution of sample means - is a
probability distribution that describes the
probability for each mean of all samples with the
same sample size n .
The Central limit theorem - states that the
sampling distribution of the mean approximates a
normal distribution with a mean of µ and a
σ
standard deviation of if the sample size N of
√n
the random samples is large enough.
Lessons and Coverage(Module Learning Competencies)

Lesson Topic/Title You’ll learn to… Estimated


No. Time
Chapter 1  illustrates a random variable (discrete and 3 weeks
Random continuous).
Variables  distinguishes between a discrete and a continuous
and
random variable
Probability
Distributions  finds the possible values of a random variable
 illustrates a probability distribution for a discrete
random variable and its properties.
 computes probabilities corresponding to a given random
variable.
 illustrates the mean and variance of a discrete
random variable
 calculates the mean and the variance of a discrete
random variable
 interprets the mean and the variance of a discrete
random variable.
 solves problems involving mean and variance of
probability distributions.
Chapter 2 Normal  identifies regions under the normal curve 1 week
Distribution corresponding to different standard normal values
 converts a normal random variable to a standard
normal variable and vice versa.
 computes probabilities and percentiles using the standard
normal table.
Chapter 3 Sampling and  illustrates random sampling. 3 weeks
Sampling  distinguishes between parameter and statistic.
Distribution  identifies sampling distributions of statistics
 (sample mean).
 finds the mean and variance of the sampling
distribution of the sample mean.
 defines the sampling distribution of the sample
mean for normal population when the variance is:
known; (b) unknown
 illustrates the Central Limit Theorem.
 defines the sampling distribution of the sample
mean using the Central Limit Theorem.
 solves problems involving sampling distributions of
the sample mean.
Lesson 4  illustrates the t-distribution. 4 weeks
 identifies percentiles using the t-table.
Estimation of  identifies the length of a confidence interval.
Parameters
 computes for the length of the confidence interval.
 computes for an appropriate sample size using the
length of the interval
 solves problems involving sample size
 determination.
Expected Skills
To do well in this module, you need to remember and do the following:
1. prepare yourself to acquire essential body of knowledge;
2. desire to work independently and learn to accurately calculate and solve mathematical problems
religiously;
3. patiently read all the contents discussed with comprehension; and follow all the steps presented on
the illustrations in answering the problems considered.

Module Map
Here is a simple map of this module.

Random Variable

Normal Distribution

Sampling and Sampling Distribution

Estimation and Parameters

LESSON 1
RANDOM VARIABLE (Discrete and Continuous)

INTRODUCTION
This module will help you understand the process of illustrating random variables as well as
distinguishing between discrete and continuous random variables. God bless and enjoy learning.
This module will help you understand the concepts on Random Variables. As you go through this
lesson, think of the following questions: How will you distinguish random variables as to discrete or
continuous? To find the answer, perform each activity.
OBJECTIVES/COMPETENCIES – ESTIMATED TIME
 illustrates a random variable (discrete and continuous).
 distinguishes between a discrete and a continuous 2 days
random variable

PRE-ASSESSMENT
Before studying this module, take this pre - assessment to find out how much you already know about the
topic covered
Activity 1– PRE-ASSESSMENT1
Direction: Choose the letter of the correct answer. Write your answer in your activity/assessment
notebook. Label your work as Q3-Lesson 1- Pre-assessment 1.
1. Discrete variable is from qualities that can be
A. Measured C. Both A and B
B. Counted D. none of these
` 2. Continuous variable is from qualities that can be
A. Measured C. Both A and B
B. Counted D. none of these
3. Which of the following is NOT a continuous variable?
A. A person’s height each year
B. A person’s weight on each birthday
C. Number of Cars finished in a factory each day
D. The volume of water in a swimming pool
4. Which of these is NOT a discrete variable?
A. The number of students absent in a class
B. The number of death per year attributed to lung cancer
C. The average amount of electricity consumed per household per month
D. The number of people who drive through a red light each hour during rush hour
5. The number of avocado produced by an avocado tree each year is a continuous variable.
A. True C. Maybe
B. False D. Cannot be determined

MOTIVATION
Well, what do you think about the pre-assessment? Do you think you did well? Let’s check how
much you already about the topic. Continue your journey and study this module to check what you already
know. Who knows, you might learn a few more new things as well.
This module was made to help you understand important concepts about random variable that you
can apply in your daily life. If you study this module carefully, you will learn the answers to all the items in
the test and a lot more! Are you ready? Let’s begin your journey.
Activity 2– Complete Me!
Direction: Complete the following table. The first one is done for you. Copy the format and write
your answer in your activity/assessment notebook. Label your work as Q3-Lesson 1- Activity 2-
Complete Me!.
Experiment Number X Possible Value of X
Two cards are drawn Sum of the numbers on 4, 5, 6, 7, 8, 9, 10,
from a deck. the cards 11, 12, 13, 14, 15,
16, 17, 18, 19, 20
Roll a pair of dice Sum of the number of
dots on the top faces
Toss a pair of coin Number of tosses until
repeatedly the coin lands head
Height of individuals Height of your father

This time, you need to reflect on the following questions because it will help you to
understand the appropriate way on illustrating and classifying a random variable.
Answer the following questions. Write your answer in your activity/assessment notebook.
Process Questions:
1. How do you find the activity/?
2. Where you able to complete the table? If yes, how? If no, why?
3. If a random variable takes on values on continuous scale it is continuous random variable but if
a random variable takes on outcomes that are countable then it is discrete randomvariable. Now,
Will you be able to classify those experiment as discrete or continuous? If yes, how? If no, why?
4. In your own words how will you differentiate between discrete and continuous random
variables?
INSTRUCTION/DELIVERY
A Random Variable is a numerical quantity that is derived from the outcomes of random
experiments.There are two types of random variables, the discrete and continuous random variables

Types of Random Variables


Discrete Continuous

A Discrete variable A Continuous variable


Is a numerical value associated with Has infinite numerical values the desired outcomes.
the desired outcomes. It is also either associated with any interval on the number line
a finite or infinite number of values system without any gaps or breaks.
a finite or infinite number of values
but countable such as whole numbers
0,1,2,3.

For example: For example:


-The number of daily - Height
admissions in a hospital - Weight
-The number of male athletes - Skull Circumference

Table 1. Examples of Random Variables


Experiment Number X Possible Values of X
1. Flip a pair of Number of heads in 4 flips of a coin
0, 1, 2, 3, 4
coin
2. Flip a pair of coin Number of tosses until the coin lands
1, 2, 3, 4, …
repeatedly tails
3. Set of Integers Integers from 2 to 6 2, 3, 4, 5, 6
4. Set of real
Real numbers from 2 to 6 2<x<6
numbers
From the example in the table above, Examples 1, 2, and 3 are all discrete while example 4 is continuous.

PRACTICE
Activity 3 – CLASSIFY ME!

This time, it’s your turn to shine. Answer the following as directed.
A. Direction: Classify as discrete or continuous random variable. Write your answer in your
activity/assessment notebook. The title of your work should be: Q3 – Lesson 1- Activity 3 –
CLASSIFY ME!
1. The speed of a bus
2. The number of voters favoring a candidate
3. The temperature in Quezon at noon time
4. The number of gold medals won by Quezon delegates in Regional Sports Competition
5. The amount of paint used in repainting a building
B. Answer the following.Write your answer in your activity/assessment notebook.
1. Ten children were asked to solve the Rubik’s cube. The time it took for them to complete the
puzzle is summarized in the table below.
Time (in min) Less than1 1-2 2-3 Greater
than 3
Number of 2 4 3 1
children
Classify what type of variable is shown.
2. Suppose that five cards are drawn from a deck, one at a time, without replacement. Let X = the
number of times a king is drawn from this experiment. What are the possible values of X? Does
the experiment represent a discrete or a continuous random variable?

EVALUATION
Direction: Look back and reflect on what you have done, then answer the following questions.
1. How do you know whether a random variable is continuous or discrete?
2. What is the difference between continuous and discrete random variables?

POST-TEST
You are now ready to take another chance to know where you are far. With your effort and
determination I believe you can do it once again. Good Luck.
Direction: Read and understand each question below. Choose your answer from the 4 given
choices. Write your answer in your activity/assessment notebook.
1. Which of the following best describe variable that can be counted?
A. Discrete C. Nominal
B. Measured D. Qualitative
2. Which of the following best describe variable that can be measured?
A. Discrete C. Nominal
B. Continuous D. Qualitative
3. Which of the following is a NOT continuous variable?
A. A person’s weight each year
B. A person’s height on each birthday
C. Number of bicycle finished in a factory each day
D. The amount of water in a pale
4. Which of the following is NOT a discrete variable?
A. The number of students present in a class
B. The number of death per year attributed to kidney failure
C. The average amount of water consumed per household
per month
D. The number of patients in a hospital each day
5. The number of coconut produced by a coconut tree each month is a continuous variable.
A. True C. Maybe
B. False D. Cannot be determined

LESSON 2
Possible Values of Random Variable
INTRODUCTION
We first learned about the notion of variables in the introduction of Algebra, and we know from our
Algebra lessons that a variable is a placeholder for real number values that can be assigned to it. Some
examples of variables includes X = number of heads or Y = number of cell phones or Z = running time to
movies.

If three coins are tossed, what numbers can be assigned for the frequency of heads that will occur? If
three cards are drawn from a deck, what number can be assigned for the frequency of face cards that will
occur? These questions may be answered using random variables which you will learn in this module.

This module for Statistics and Probability will help you understand the process on how to find the
possible values of random variables. This module has pre-test and post-test for you to track your progress as
you go on with the lesson. The pre-test will determine your prior knowledge of the lesson while the post-test
will measure your learning after you work with this module. Read carefully the situations and examples
given for you to understand the lesson well.

OBJECTIVES/COMPETENCIES – ESTIMATED TIME


Find the possible values of a random variable 2 days

PRE-ASSESSMENT

Before studying this module, take this pre - test to find out how much you already know about the
topic covered.

Activity 1– PRE-ASSESSMENT1
Direction: Choose the letter of the correct answer and write it in your activity/assessment
notebook.
1. If two coins are tossed, which is NOT a possible value of the random variable for the number of
heads?
A. 0 B. 1 C. 2 D. 3
2. Which of the following is NOT a true statement?
A. The value of a random variable could be zero.
B. Random variables can only have one value.
C. The probability of all the value of a random variable could be zero.
D. The sum of all the probabilities in a probability distribution is always equal to one.
3. You decided to conduct a survey of families with two children. You are interested in counting
the number of boys (out of 2 children) in each family. Is this a random variable? If it is, what are
the possible values?
A. Yes, it is a random variable and its value can be 1 or 2.
B. Yes, it is a random variable and its value can be 0, 1 or 2.
C. Yes, it is a random variable and its value can be 2 or 4.
D. No, it is not a random variable since it is not random.
4. In a local community, a couples were asked the questions “Are you satisfied with the work of
the current president?” If the husband and the wife both said “yes”, the response is written as
YY. If the husband said yes and the wife said “no”, the response is YN. Let X = the number
of “yes” responses. , what are the possible values of the random variables?
A. 0, 1, 2 B. 1, 2, 3 C. 2, 3, 4 D. 1, 1, 2
5. A quantity resulting from an experiment by chance, can assume a different values is called
A. Random Experiment C. Random Variable
B. Random Sample D. Random process

How did you find the pre-test? Have you got the correct answers? If yes, Very Good! But, if you
failed to get it right don’t be hesitant to learn new concepts because it only indicates that you need to ponder
yourself by exploring this module. By that, you will acquire the proper way on how to deal with those
situations. The next time you encounter those scenarios, things would be easy for you.

Be focused on the details that will be presented and will be discussed in this module in order for you
to take into the proper procedure in answering those problems.
Let us now discuss this Lesson.
.MOTIVATION
Activity 2– Answer Me!
Direction: Read and analyze the problem below. Answer the questions and write your answer in
your activity/assessment notebook.

l phone, we assign the number 0; if there is 1 defective cell phones, we assign the number 1; if there are

Read and analyze the given situation below.


Illustration:
Let D represent the defective cell phone and N represents the non – defective cell phone. If we let
X be the random variable representing the number of defective cell phones, show the values of the
random variable X? Complete the table below to show the values of the random variable.
Value of the Random Variable X
Possible Outcomes This time,
(number of defective cell phones)
you need to
reflect on the
following
questions
because it
will help you
to understand the appropriate way on finding the possible values of a random variable. Answer
the following questions. Write your answer in your activity/assessment notebook.
1. How do you find the activity?
2. Were you able to complete the table? If yes, how? If no, why?
3. Will you be able to find the values of the random variable? If
yes, how? If no, why?
4. In your own words how will you describe a random variable?
5. How do you find the possible values of a random variable?

INSTRUCTION/DELIVERY
A Random Variable is a numerical quantity that is derived from the outcomes of random
experiments.
The random variable in the activity above is a discrete random variable because the set of possible outcomes
is countable. The possible values of random variable X are 0, 1, 2, and 3.
Example 1. Suppose three coins are tossed. Let Y be the random variable representing the number of tails
that occur. Find the values of the random variable Y.
Steps Solution
1. Determine the The sample space for this experiment is:
sample space. Let S = {TTT, TTH, THT, HTT,
H represent head HHT, HTH, THH, HHH}
and T represent tail.
2. Count the number of Possible Value of the Random variable
tails in each outcome Outcomes Y (Number of tails)
in the sample space TTT 3
and assign this TTH 2
THT 2
number to this
HTT 2
outcome.
HHT 1
HTH 1
THH 1
HHH 0

So, the possible values of the random variable Y are 0, 1, 2, and 3.


Example 2.
Two balls are drawn in succession without replacement from a box containing 5 red balls and 6 blue balls.
Let Z be the random variable representing the number of blue balls. Find the values of the random variable
Z.
Steps Solution
1. Determine the The sample space for this
sample space. Let experiment is:
B represent the blue S = {RR, RB, BR, BB}
ball and R represent
the red ball.
2. Count the number of Possible Value of the
blue balls in each Outcomes Random
outcome in the sample variable Z
space and assign this (Number of
number to this blue balls)
outcome. RR 0
RB 1
BR 1
BB 2

So, the possible values of the random variable Z are 0, 1 and 2.


PRACTICE
This time, it’s your turn to shine.
Activity 3 – Problem Solving!

Direction: Answer the problem below. Write your answer in your activity/assessment notebook.
The households of a local community were surveyed about the number of occupants who
are working. It was found out that 25 households have one occupant working, 18 have two
occupants working, 12 have three occupants working, and 5 have four occupants working. Let X
be the number of occupants working from a randomly selected household. What are the possible
values for the random variable?
EVALUATION
Direction: Answer the following as directed. Write your answer in your activity/assessment
notebook.
1. How do you find the values of a random variables?
2. How is this variable, as described in Algebra, similar to a random
variable? How do they differ?

POST-TEST
You are now ready to take another chance to know where you are so far. With your effort and
determination I believe you can do it once again. Good Luck.
Direction: Choose the letter of the correct answer. Write your answer in your activity/assessment
notebook.
1. If two coins are tossed, which is not a possible value of the random variable for the number of
tails?
A. 0 B. 1 C. 2 D. 3
2. Which of the following is a true statement?
A. The value of a random variable could not be zero.
B. Random variables can only have one value.
C. The probability of all the value of a random variable could be Zero.
D. The sum of all the probabilities in a probability distribution is not equal to
one.
3. You decide to conduct a survey of families with two children. You are interested in counting the
number of girls (out of 2 children) in each family. Is this a random variable and if it is, what
are the possible values?
A. Yes, it is a random variable and its value can be 1 or 2.
B. Yes, it is a random variable and its value can be 0, 1 or 2.
C. Yes, it is a random variable and its value can be 2 or 4.
D. No, it is not a random variable since it is not random.
4. In a local community, a couples were asked the questions “Are you satisfied with the work
of the current president?” If the husband and the wife both said “yes”, the response is
written as YY. If the husband said yes and the wife said “no”, the response is YN. Let
X = the number of “yes” responses. , what are the possible values of the random
variables?
A. 0, 1, 2 B. 1, 2, 3 C. 2, 3, 4 D. 1, 1, 2
5. A set of numerical values assigned to a sample space is called
A. Random Experiment C. Random Variable
B. Random Sample D. Random process

LESSON 3
Constructing Probability Distribution

INTRODUCTION
You have learned that the likelihood of winning in a lotto draw, number of winnings in a gamble,
number of heads that comes out in a toss of a coin and other game of chance can be estimated using
probability. But do you know that it is not only for game of chance? We also use this in business,
economics, and engineering and other real-life situations.

OBJECTIVES/COMPETENCIES – ESTIMATED TIME


 illustrates a probability distribution for a discrete random variable 3 days
and its properties.
 computes probabilities corresponding to a given random variable.

PRE-ASSESSMENT
Before starting with the lesson of this module, answer the following questions first. This will
determine what you already know about the topic.
Activity 1– PRE-ASSESSMENT1
Direction: Choose the letter that corresponds to the letter of your answer. Write your
answer in your activity/assessment notebook. Entitle your work asQ3 – Lesson 3 – Activity 1 –
Pre-Assessment 3.
1. What is the sum of the probabilities of all values of the random variable?
A. ∑ (P ( X ) )= 0 C.∑ ( P ( X ) ) = 1
B. ∑ ( P ( X ) ) = 1/10 D.∑ ( P ( X ) ) = 10
2. If two balls are drawn in succession without replacement from an urn containing 5 red balls
and 6 blue balls. If the value of the random variable represents the number of blue balls.
How many outcomes are possible ?
A.2 B. 4 C. 6 D. 8
3. Which of the following cannot be the value of probability of the random variable ?
A. 1.01 B. 0 C. 1/4 D. 1/2
4. Which of the following can serve as the values of a probability distribution ?
A. P(1) = 0.42, P2) = 0.31, P(3)= 0.37
B. P(1) = 9/14, P2) = 4/14, P(3)= 1/14
C. P(1) = 0.08, P2) = 0.12, P(3)= 1.03
D. P(1) = 10/33, P2) = 12/33, P(3)= 10/33
x
5. If P(X)= , what are the possible values of X for it to be a probability distribution?
6
A.0,2,3 B.1,2,3 C. 2,3,4 D.1,1,2
Well, what can you say about the pre-test ? Do you think you did well ? Let’s continue our
journey.
MOTIVATION
Activity 2– Answer Me!
Direction: Answer the questions below as directed. Write your answer in your
activity/assessment notebook. Entitle your work asQ3 – Lesson 3 – Activity 12– Answer Me!.
Number of Defective Computer
A. In a computer laboratory, the teacher wants to find out if there is a defective computer.
Supposed three computers are tested at random, she asks one of her CSS students to list all the
possible outcomes, such that D represents the defective computer and N represents the non-
defective computer. Let X be the random variable for the number of defective computers.
Find the value of the random variable X.
B. List the sample space in the given experiment. Let D represent the defective computer and N
represent the non-defective computer. Count the number of defective computers in each outcome
in the sample space and assign this number to this outcome. For instance , if you list NND the
number of defective computer is 1.
Value of the Random Variable X
Possible Outcomes
(number of defective computers)

C. Assign probability values P(X) to each value of the random variable


Number of Defective Computers X Probability P(X)
D. Construct a histogram for a probability distribution. Remember that a histogram is a bar graph.
Plot the values of the random variable along the horizontal axis, then plot the probabilities along
the vertical axis.
This time, you need to reflect on the following questions. Answer the following questions.
Write your answer in your activity/assessment notebook.
1. How did you find the activity?
2. How many outcomes did you obtained?
3. What are the values of the random variable?
4. How will you find the probability to each value of a random variable?
5. In the preceding probability distributions, what do you notice about the probability of each
value of the random variable?
6. What is the sum of the probabilities of all values of the random variable?

INSTRUCTION/DELIVERY
A random variable is a set whose elements are the numbers assigned to the outcomes of an
experiment. It is usually denoted by uppercase letters such as X, whose elements are denoted by lower
case letters x1, x2, x3 and so on.
Probability distribution of a discrete random variable specifies the probability of each possible value
of the random variable. The distribution functions of discrete random variables are concentrated as a mass
for a particular value, and generally known as Probability Mass Function.

Properties of discrete probability distribution


1. The probability of each value of the random variable must be between or equal to 0
and 1. In symbol 0 < P(X) < 1.
2. The sum of all the probabilities of all values of the random variable must be equal to
1. In symbol, we write it as ΣP(X) = 1

Probability Histogram
We can represent a probability distribution graphically by constructing a type of bar graph called a
probability histogram. This is constructed by displaying the possible distinct values of the random variable
along a horizontal axis. Above each value x of the random variable, we draw vertical bar having height
equal to the probability P(x).
Ex. 1. Number of Green Balls
Write the probability distribution of a random variable Y representing the number of green balls
when 2 balls are drawn in succession without replacement from a jar containing 4 red and 5 green balls.
Construct a histogram for this probability distribution.
Steps Solution
a.Determine the sample space. Let R represent The sample space for this experiment is:
the red ball and G represent the green ball. S= { RR, RG, GR, GG}
b.Count the number of green balls in each Possible Value of the Random
outcome in the sample space and assign this Outcomes Variable Y (No. Of
number to this outcome. Green Balls)
RR 0
RG 1
GR 1
GG 2
c.Assign probability values P(Y) to each value of Number of Green Probability P(Y)
the random variable. Balls Y
-There are 4 possible outcomes, if no green ball
occurs once, the probability that we shall assign to
0 1/4
the random variable 0 is ¼
-If 1 green ball occurs two times, the probability
that we shall assign to the random variable 1 is 2/4
or ½.
1 2/4 or ½
-If 2 green balls occur once, the probability that we
shall assign to the random variable 1 is ¼.

2 ¼

Histogram

Construct a histogram for this probability distribution. A histogram is a bar graph. To construct a
histogram for a probability distribution, follow these steps.
Plot the values of the random variable along the horizontal axis.
In plotting the probabilities along the vertical axis, you can change fractions to decimals (ex. ¼=0.25 and
½= 0.5)
0.6
0.4
P(Y)

0.2
0
0 1 2

No. of Green Balls (Y)

Ex. 2.Write the probability mass function of the random variable X defined by the outcomes of rolling a
fair die and construct its corresponding histogram.
a)Since a die is fair, each of the 6 outcomes has a probability 1/6 or approximately 0.17, thus, the
probability mass function is:
X 1 2 3 4 5 6
P(X) 1/ 1/6 1/6 1/6 1/6 1/6
6
b) Histogram
0.18

0.16

0.14

0.12

0.10
P(X)

0.08

0.06

0.04

0.02

0.00
1 2 3 4 5 6
Ex. 3.Let X be a random variable giving the number of girls in a randomly selected three-child family.
Assuming that boys and girls are equally likely , construct the probability distribution of X and its
corresponding histogram.
a.)

Possible Value of the Random Variable X


Outcomes
(No. of Girls)

BBB 0

BBG 1

BGB 1

GBB 1

BGG 2

GBG 2
b) No. of Girls GGB 0 1 2 2 3

Probability GGG1/8 or 3/8 or 3 3/8 or 1/8 or 0.125


P(X) 0.125 0.375 0.375
ΣP(X)= 1/8 +3/8 +
3/8 + 1/8 = 8/8 =1
c)Histogram
0.4
0.35
0.3
0.25
P(X)

0.2
0.15
0.1
0.05
0
0 1 2 3

No. of Girls (X)

4.
X 1 5 7 8 9
P(X) 1/3 1/3 1/3 1/3 1/3

Σ P(X)= 1/3 + 1/3 +1/3 +1/3 +1/3=5/3


This is not a probability distribution because ΣP(X) ≠ 1

X 1 3 5 7
5. P(X) 0.35 0.25 0.28 0.12
Σ P(X)= 0.35 + 0.25 + 0.28 + 0.12 =1
This is a probability distribution because ΣP(X) =1
PRACTICE
Direction: Use the procedure that you have learned to solve the following problems involving
probability distribution of a discrete random variable.
A. Construct the probability distribution for the random variables described in each of the
following situations .Draw the corresponding histogram for each probability distribution.
1. In a National Achievement Test, suppose three test booklets are tested at random. Let D
represent the defective test booklet and let N represent the non-defective test booklet. Let X be
the random variable representing the number of non-defective test booklets.
2. An experiment consists of asking three Grade 10 students at random if they prefer to choose
Academic track or TVL track in the senior high school. Use A for Academic and T for TVL. Let Y
represent the random variable representing the number of TVL track responses.
3. Suppose that a coin is to be tossed four times, and let X represent the number of tails that
occur.
4.Using the sample space for rolling two dice , construct a probability distribution for the random
variable X representing the sum of the numbers that appear.
B. Determine whether the distribution represents a probability distribution. Explain your answer.
1. X 1 5 7 8
P(X) 1/4 1/8 1/4 1/8

2. X 0 2 3 4 6
P(X) 1/6 1/6 1/6 1/3 1/6

3. X 1 3 5 7
P(X) 0.35 0.25 0.22 0.12

4. P(1)= 0.42, P(2)= 0.31, P(3)= 0.37


5. P(1)= 10/33, P(2)= 1/3, P(3)= 12/33

EVALUATION
Direction: Look back and reflect on what you have done, then answer the following questions.
Write your answer in your activity/assessment notebook.
1. Why should the sum of the probabilities in a probability distribution always equal to 1?
2. What is the shape of most probability distributions? Why do you think so ?
POST-TEST
You are now ready to take another chance to know where you are so far. With your effort and
determination I believe you can do it once again. Good Luck.
Direction: Shade the circle that corresponds to the letter of your answer.
1.What is the sum of the probabilities of all values of the random variable?
A P(X) = 1 C. P(X) = 0
B P(X) = 1/10 D. P(X) = 10
2.If two balls are drawn in succession without replacement from an urn containing 3 red balls
and 5 blue balls. If the value of the random variable represents the number of red balls.
How many outcomes are possible ?
A.8 B. 6 C. 4 D. 3
3.Which of the following cannot be the value of probability of the random variable ?
A. 1.01 B. 0 C. 1/4 D. ½
4.Which of the following can serve as the values of a probability distribution ?
A. P(1) = 0.42, P2) = 0.31, P(3)= 0.37
B. P(1) = 0.08, P2) = 0.12, P(3)= 0.83
C. P(1) = 9/14, P2) = 5/14, P(3)= 1/14
D. P(1) = 10/33, P2) = 1/3, P(3)= 12/33
X
5.If P(X)= , what are the possible values of X for it to be a probability distribution?
8
A.0,2,3,4 B.0,1,2,3 C. 1,3,4 D.1,2,3

LESSON 4
Illustrating and Calculating Mean and Variance of Discrete Random
Variable
INTRODUCTION
If you took an examination, do you sometimes think of where would be your score lie? Would it be
below or above the average score? Afterwards, you would like also to know how far you are from the
average score. Finally, you would ask yourself what is the meaning of those locations from the center or
mean of the probability distribution? If your interest caught by these questions, this learning module is
design for you.
In this module, you will understand the process on how to illustrate the mean and variance of a
discrete random variable. Similarly, calculating the mean and the variance of a discrete random variable will
be discussed. You may find this module as piece of good fortune in acquiring brand new learning.
OBJECTIVES/COMPETENCIES – ESTIMATED TIME
 illustrates the mean and variance of a discrete
random variable 3 days
 calculates the mean and the variance of a
discrete
random variable

PRE-ASSESSMENT
Before studying this module, take this pre- test to find out what you already know about the topics covered.
Activity 1– PRE-ASSESSMENT1
Direction: Choose the letter of the correct answer. Write your answer in your activity/assessment
notebook. Entitle your work as Q3 – Lesson 4 – Activity 1 – Pre-Assessment 4.
1. The appropriate formula in finding the mean of discrete random variable is
A. E ( x )=μ x =∑ x ∙ p(x) C. E ( x )=μ x =∑ x− p( x )
B. E ( x )=μ x =∑ x+ p ( x) D.E ( x )=μ x =∑ x ∙ p(x)2

2.What formula is used to find the variance of discrete random variable?


A. σ 2x =∑ (x+ μ)2 ∙ p( x ); for all possible values of x
B. σ 2x =∑ (x−μ)2 ∙ p (x); for all possible values of x
C. σ 2x =∑ x ∙ p(x ); for all possible values of x
D. σ 2x =∑ ( P( x)+ μ)2 ∙ x ; for all possible values of x
3.Which statement is true about the standard deviation of a
discrete random variable?
A. deals about the average or center of location of the probability distribution.
B. It is obtained by multiplying the x values and its corresponding probability.
C. It is the square root of the variance of discrete random variable.
D. It is the summation of the product of the square of the difference of x and its probability.
4. In a recent Barangay Basketball League, each player went to free throws 2 times. The number
of free throws made by each player is described by the following probability distribution.
Number of free
throws, Probability,P(X)
x
0 0.20
1 0.45
2 0.35
What is the mean of the probability distribution?
A. 1.00 B. 1.15 C. 2.00 D. 2.25
5. The number of qualified voters living in household on a randomly selected subdivision block is
described by the following probability distribution.

Number of qualified voter/s,


1 2 3 4
x
Probability, P(X) 0.25 0.50 0.15 0.10
What is the variance and standard deviation respectively of the probability distribution?
A. 0.99 and 0.50 C.0.80 and 0.79
B. 0.89 and 0.62 D.0.79∧0.89
This time, be focused on the details that will be presented and will be discussed in this module in
order for you to take into the proper procedure in answering those problems. Let us now discuss the lesson.

MOTIVATION
Activity 2– Answer Me!
Direction: Read and analyze the situation given below: Write your answer in your
activity/assessment notebook. Entitle your work asQ3 – Lesson 4 – Activity 2 – Answer Me!.
Mr. Cresente Umali, a Mathematics teacher, regularly gives a formative assessment
composed 5 multiple choice items. After the assessment, he used to check the probability
distribution of the correct responses and the data is presented below:
Test Probability ,
Item
p( X )
X
0 0.03
1 0.05
2 0.12
3 0.30
4 0.28
5 0.22
Based on the data above, observe, analyze and answer the following questions:
1. Is the probability of x lies between 0 and 1?_____________________
2. What is the sum of all probabilities of x?_______________________
3. Is there a negative probability? Is it possible to have a negative probability?
________________________________________________
4. What is the average or mean of the probabilities of discrete random variable?
__________________________________________________
5. What are the variance and the standard deviation of the probability distribution from the
data above?_______________________________
This time, you need to reflect on the following questions because it will help you to understand the
appropriate way on illustrating and calculating the mean, variance and standard deviation of a discrete
random variable.
Answer the following questions:Write your answer in your activity/assessment notebook.
1. How did you find the given activity?
2. Were you able to get the correct answer for each question? If yes, Very Good! If not, which
question/s you find difficult?
3. What mathematical skills help you to answer all 5 questions?
4. Do you think the concepts that you will acquire in this module would be helpful to solve real-
life problems?

INSTRUCTION/DELIVERY
The Expected or mean value of a discrete random variable x, denoted by x, is computed by first
multiplying each possible x value by the probability of observing that value and then adding the resulting
quantities. Symbolically, E ( x )=∑ xP( x ).

Using the given data, multiply each x value by its probability and add the results to getμx.

Test Item, Probability ,


x ∙ P( X )
x P( X )
0 0.03 0
1 0.05 0.05
2 0.12 0.24
3 0.30 0.90
4 0.28 1.12
5 0.22 1.10

E ( x )=μ x =∑ x ∙ P ( X )=3.41
So, it indicates that the expected value or mean of the random variable is 3.41.
For question 5, you will need to understand the process on computing the variance and the standard
deviation of the discrete random variable. In order to compute those, always remember this;
The Variance of a Discrete Random Variable X, denoted by σ 2x is computed by first subtracting the
mean from each possible x value to obtain the deviations, then squaring each

deviation and multiplying the result by the probability of the corresponding x value, and then finally adding
these quantities. In symbol,

The Standard Deviation of x, denoted by

σ x , is the square root of the Variance. In symbol,


From the data generated.
μ x =∑ x ∙ P ( X )=3.41 ∑ ¿ ¿

The
Test Item, Probability
x ∙ P( X ) x−μ ¿ ¿ Variance
x P( X ) is
1.5819,
0 0.03 0 -3.41 11.6281 0.3488 and
1 0.05 0.05 -2.41 5.8081 0.2904
The
2 0.12 0.24 -1.41 1.9881 0.2386 Standard
3 0.30 0.90 -0.41 0.1681 0.0504 Deviation
is
4 0.28 1.12 0.59 0.3481 0.0975
√ 1. 5819 ,
5 0.22 1.10 1.59 2.5281 0.5562 and it is
equivalent to σ =¿1.26
Therefore, the correct answers for question 5 are 1.5819 and 1.26 respectively.
For uniformity of the answers, expected value or mean and standard deviation are both expressed into two
decimal places while variance is up to four decimal places to lessen rounding error.
Let us take this example,
Suppose that a coin is tossed twice so that the sample space is S = {𝐻𝐻, 𝑇𝐻, 𝐻𝑇, 𝑇𝑇}. Let X represents the
“number of heads that can come up”, Based on the prepared discrete probability distributions of the random
variable X.
Outcome or Sample Point HH HT TH TT
X 2 1 1 0
Illustrate and calculate the mean, variance and standard deviation.
Applying the concepts from the activity given above, let us complete the table below.
x P( X ) x ∙ P(X ) x−μ ¿ ¿
0 ¼ or 0.25 0 -1 1 0.25
1 ½ or 0.5 0.5 0 0 0
2 ¼ or 0.25 0.5 1 1 0.25
μ x =∑ x ∙ P ( X )=1 ∑ ¿¿
The expected value or mean is 1.
The Variance is 0.50, and
The Standard Deviation is √ 0 .5 0, and it is equivalent to σ =¿0.71.

We have seen that the Expected value E(x) is just the average or mean (µ) of random
variable x. It’s sometimes called a “weighted average” because more frequent values of X are
weighted more highly in the average. It’s also how we expect X to behave on-average over the
long run. The mean of a random variable X is a measure of the central location of the distribution
of X. If we are summarizing features of the distribution of X, it is clear that location is not the only
relevant feature. The second most vital feature is the spread of the distribution.

If values of X is closer to its mean, μ X are very likely and values further away from μ X have
very small probability, then the distribution of X will be closely concentrated around μ X. In this
case, the spread of the distribution of X is small. On the other hand, if values of X take some
distance from its mean μX are likely, the spread of the distribution of X will be large.
These ideas head to the most significant measure of spread, the variance, and a closely
related measure, the standard deviation.

You, as a student have met the concepts of variance and standard deviation when
summarizing data. These were the sample variance and the sample standard deviation. The
difference here is that we are referring to properties of the distribution of a discrete random
variable.

Again, the variance of a discrete random variable X is defined by

where the sum is taken over all values of x for which P(x) > 0 but less than 1. So, the variance of X
is the weighted average of the squared deviations from the mean μ, where the weights are given
by the probability function P(x) of X.

The standard deviation of X is defined to be the square root of the variance of X. That is,

Because of this definition, the variance of X is often denoted by


σ2x.
In another way around, the standard deviation is the more tangible of the two measures,
since it is in the same units as X. For example, if X is a random variable measuring lengths in ft,
then the standard deviation is in feet (ft), while the variance is in square ft (ft 2).

Unlike the mean, there is no simple direct interpretation of the variance or standard
deviation. The variance is parallel to the moment of inertia in physics, but that is not necessarily
widely understood by students. What is important to understand is that, in relative terms:
 a small standard deviation (or variance) means that the distribution of the random variable
is concentrated narrowly around the mean
 a large standard deviation (or variance) means that the distribution is spread out, with some
chance of observing values at some distance from the mean.
Remember that the variance cannot be negative, because it is an average of squared
quantities. This is appropriate, as a negative spread for a distribution does not make sense.
Hence, variance is greater than 0 and standard deviation also greater than 0 always.
PRACTICE
This time, it’s your turn to shine. Answer the following problems.
1. The number of shoes sold per day at a retail store is shown in the table below. Find the mean,
variance, and standard deviation of this distribution,
X 19 20 21 22 23
0. 0.
P(X) 0.1 0.4 0.1
2 2

2. Suppose that a coin is to be tossed four times, and let X represents “the number of TAILS that
can come up”. Find the mean, variance, and standard deviation of this distribution.
3. The number of patients seen in the Emergency Room in any given hour is a random variable
represented by x. The probability distribution for x is:

X 10 11 12 13 14
0. 0.
P(X) 0.2 0.2 0.1
4 1
Compute for the mean, variance and standard deviation.

EVALUATION
Direction: Answer the question below. Write your answer in your activity/assessment
notebook.
1. Life is not measured by the number of breaths we take, but by the moments that take our breath
away and by the chances we bravely face. Please write your own thoughts.

POST-TEST
In order to determine your learning in this module, answer the following. Please shade the
letter of the correct answer.

1. Let X be a random variable defining number of students getting 95 above grade. Find the
expected value of X from the given table.
X 0 1 2 3
P(X) 0.2 0.1 0.4 0.3
A . 1 . 3
C. 1.9 D. 2.3
2. A Grade 11 – HUMMS researcher surveyed the households in Brgy Quipot, Tiaong Quezon.
The random variable X represents the number of college graduates in the households. The
probability distribution of X is shown below.
X 0 1 2
P(X) 0.25 0.5 0.25
Find the values of variance and
A. 0.3 and 0.51 respectively
B. 0.4 and 0.61 respectively
C. 0.5 and 0.71 respectively
D. 0.6 and 0.81 respectively

3. Which of the following statements is TRUE about the interpretation of the values of variance
and standard deviation?
A. A small value of variance or standard deviation indicates that the distribution of the
discrete random variable is closer about the mean.
B. A large value of variance or standard deviation indicates that the distribution of the
discrete random variable is closer about the mean.
C. A small value of variance or standard deviation indicates that the distribution of the
discrete random variable takes some distance from the mean.
D. All of the above.
4. In 50 items test, Miss Santos, a mathematics teacher claimed that most of the students’ scores
lie closer to 40. In this situation, score of 40 is considered as,
A. Variance
B. Standard Deviation
C. Expected Value or Mean
D. Median
5. Which of the following statement describes variance of a
discrete random variable?
A. It is a small value of variance or standard deviation indicates that the distribution of the
discrete random variable is closer about the mean.
B. It is the product of mean and the square of the probability distribution of a discrete random
variables.
C. It is obtained by getting the summation 0f the product of the square of the difference
between the value of X and expected value times its corresponding probability
D. All of the above

LESSON 5
Interpreting the Mean and Standard Deviation of Discrete Random
Variable

INTRODUCTION
Are you interested in buying lotto tickets? How about making a bet in a roulette wheel or
taking a chance in raffle tickets? Do you want to know how much gain will you have if you will win
in games of chance or how much money will be wasted if you lost on a bet? Are you planning to
invest money in a life insurance company? If you had thought of the following questions this
module is for you.
In this module, you will learn how to compute the probability or the chance of winning and
losing. You will apply the formula for the expected mean to solve real life problems involving mean
and variance of probability distributions. Good luck and happy learning.
OBJECTIVES/COMPETENCIES – ESTIMATED TIME
 interprets the mean and the variance of a
discrete 3 days
random variable.
 solves problems involving mean and variance
of
probability distributions.

PRE-ASSESSMENT
Before studying this module, take this simple test to determine what you already know
about the topic covered.

Activity 1– PRE-ASSESSMENT1
Direction: Choose the letter of the correct answer. Write your answer in yout activity/assessment
notebook.
1. You buy one ₱500.00 raffle ticket for a prize of new car (Toyota Wigo) valued at ₱675, 000.00.
Two thousand tickets are sold. If X denotes the net gain from the purchase of a randomly selected
ticket, which of the following tables of values shows the probability distribution of X?
A. C.
x 675,000 -500 x 674,50 -500
P(x) 1 1999 0
2000 2000 P(x) 1 1999
2000 2000
x 674,50 500
x 675,00 500 D
B. 0
0 .. P(x) 1 1999
P(x) 1 1999
2000 2000
2000 2000
2. What is the mean or the expected value of item number 1?
Refer to problem number 1)
A. - 162.5 B. -162.25 C. 162.25 D. 162.5

3. One thousand tickets are sold for ₱10.00 each. One ticket will win ₱2,000.00, two tickets will
win ₱1,000.00 each and four tickets will win ₱500.00 each. What is the probability of
winning any amount in the purchase of one ticket?
1 2 5 7
A. B. C. D.
1000 1000 1000 1000
4. A roulette wheel in a fiesta carnival has the numbers 1 through 50. If you bet ₱5.00, you will
have a chance to win a kitchen utensil worth ₱200.00. How much the organizer will earn if
100 games will be played?
A. ₱2,000.00 B. ₱3,000.00 C. ₱4,000.00 D. ₱5,000.00

5. A life insurance company will sell a ₱500,000.00 five-year term life insurance policy exclusive
for police enforcers for a premium
A B C D
of ₱1,000.00.B.Find
A. ₱550.00 the expected
₱650.00 value to theD.
C. ₱750.00 company
₱850.00of a single
This module was made to help you understand important concepts about mean or expected
value of discrete random variable that you can apply in your daily life. If you study this module
carefully, you will learn the answers to all the items in the test and a lot more! Are you ready? Let’s
begin your journey.

MOTIVATION
Activity 2– GAME OF CHANCE
Direction:Read the conversation below and answer the questions that follow.Write your
answer in your activity/assessment notebook.
Questions:
1. If you are Cardo, would you buy a raffle ticket? Why?
______________________________________________________________
_____________________________________________________________.
2. How would you describe Romulo as a friend?
________________________________________________________________________
____________________________________________________
3. If Cardo decided to buy one ticket, what is the probability that he would win the prize if 500
tickets were sold? ________ What is the probability that Cardo will lose the bet?_________
4. How much money will Cardo gain if he wins the prize? _______. How much money will be
wasted if he will not win the prize? ____________________
5. If you were Cardo, will you buy a ticket? __________________________.
This time, you need to reflect on the following questions because it will help you to
understand the concept of the mean and variance of a discrete random variable. Your honest
answer would be a big help for the success of your journey on this module.
Direction: Answer the process questions below and write your answer in your
activity/assessment notebook.
1. How do you find the activity?
2. Were you able to answer all five questions? If yes, what helped you to answer each question? If
no, why were you not able to answer those questions?
3. Have you experienced a situation where you need to decide to take a chance? If yes, reflect on
the factors which affect your decision.
4. What mathematical concepts or skills will help Cardo make his decision? How important these
concepts/skills for Cardo to decide and choose the best option?
5. Do games of chance really help individuals with financial problems? Explain.

INSTRUCTION/DELIVERY
Mean or Expected Value and Standard Deviation of a Discrete
Random Variable.
Expected value is exactly what you might think, it means intuitively: the return you can
expect for some kind of action. It informs about what to expect in an experiment "in the long run",
after many trials.
In the study of probability distribution the mean of possible values of a discrete random variable,
given by their respective probabilities, is known as the expected value, usually represented by E ( x ) 
or μ X .To compute for the mean of a discrete random variable, we use the formula
E ( x )=μ x =∑ [ x i ∙ P(x i) ]
where x iis the value of the random variable for outcome i, μ x is the mean of random variable X,
and P(x i)is the probability that the random variable will be outcome i.

EXAMPLE Senior citizens of a particular barangay organize a Christmas raffle bonanza. One
1 thousand raffle tickets are sold for₱50.00 each. Each one has an equal chance of
winning. The first prize is a 32-inch LED TV worth ₱10,000.00, the second prize is an electric oven
worth ₱5,000.00 and the third prize is a grocery pack worth ₱2,500.00. Let X denotes the net gain
from the purchase of one ticket.
a. Construct the probability distribution of X.
b. Find the probability of winning any amount in the purchase of one ticket.
c. Find the expected value of X, then interpret.
Solution:
a. If a ticket is selected as the first prize winner, the net gain to the purchaser is ₱10,000.00 less
the ₱50.00 that was paid for the ticket, hence x 1 = 10,000 – 50 = 9950. There is one such ticket,
1
so the probability of gaining 9950 is one out of one thousand tickets or which is equal to
1000
0.001. If it is selected as the second prize winner, the net gain to the purchaser is ₱5,000.00 less
1
the ₱50.00, hence x 2= 5,000 – 50 = 4950 with a probability of or 0.001. If it is selected as
1000
the third prize winner, the net gain to the purchaser is ₱2,500.00 less the ₱50.00, hence x 3=
1
2,500 – 50 = 2450 with a probability of or 0.001. But if the ticket was not selected to win the
1000
prize, the purchaser lost ₱50.00 that was paid for the ticket, hence x 4= – 50 with a probability of
997
997 out of 1000 tickets or which is equal to 0.997.
1000
Below is the probability distribution:
xi 9950 4950 2450 -50
P( x i) 0.001 0.001 0.001 0.997

b. If we let W denote the event that a ticket is selected to win one of the prizes. Using the table
The probability of winning the first prize is 0.001.
The probability of winning the second prize is 0.001.
The probability of winning the third prize is 0.001.
Hence P(W ) = 0.001 + 0.001 + 0.001 = 0.003
c. Using the formula of expected value or mean.
E ( x )=9950 ( 0.001 ) +4950 ( 0.001 ) +2450 ( 0.001 ) +(−50)( 0.997)
E ( x )=−32.50

The negative value means one loses money on the average. In particular, if someone
were to buy tickets repeatedly, and although he would win now and then, on average he would
lose thirty two pesos and fifty cents per ticket purchased.
You may now go back to the problem of Cardo and apply what you have learned from
example 1.
To answer item number 3: If Cardo decided to buy one ticket, what is the probability that he
would win the prize if 500 tickets were sold? What is the probability that Cardo will lose the bet?

Solution:
1
You need to compute for the probability of the two events. Cardo will have or 0.002
500
499
chance to win the prize and or 0.998 chance to lose it. It is also clear that he will gain
500
₱15,000.00 less ₱100.00 or ₱14,900.00 if he wins the prize and his ₱100.00 will be wasted if he
will lose the prize.
The concept of expected value is also applicable to the insurance industry, as illustrated by the
example below.
A life insurance company will sell a ₱250,000.00 one-year term life insurance
EXAMPLE
policy for members of armed forces of the Philippines for a premium of ₱200.00.
2
Find the expected value to the company of a single policy if a member of the
armed forces police has a 99% chance of surviving one year?
Solution:
Let X denotes the net gain to the company from the sale of one policy. There are two
possibilities: the insured person lives the whole year or the injured person dies before the year is
finished. Applying the “income minus expenses” principle, in the first case the value of X = 200 –
0; in the latter case it is 200 – 250,000 = - 249,800. Since the probability in the first case is 99.98%
or 0.9998 and in the latter case is 1 – 0.9998 = 0.0002, the probability distribution for X is:
x 200 -249,800
P( x ) 0.9998 0.0002
Therefore
E ( X ) =∑ xP ( x ) =200 ( 0.9998 )+ (−249,800 ) ( 0.0002 )

E ( x )=¿150

Interpretation:
Sometimes the company may lose a large amount of money on a policy, but typically gains
₱200.00, which by the computation of E(X) gives a net of ₱150.00 on the average per policy sold.

PRACTICE
LET’S PLAY A GAME
Direction: Use the formula you have learned to solve the following real life problems (games of
chance) involving mean and variance of a discrete random variable. Use the back of this page or a
separate sheet of paper for your computations, use of calculator would be a help. Enjoy and
happy working . Write your answer in your activity/assessment notebook.
Game of Chance 1. (Raffle Tickets)
Two thousand tickets are sold for ₱20.00 each. One ticket will win ₱10,000.00, two tickets will win
₱5,000.00 each and three tickets will win ₱1,000.00 each. Let X denotes the net gain from the
purchase of a randomly selected ticket.
a. Construct the probability distribution of X.
b. Find the probability of winning any amount in the purchase of one ticket.
c. Find the expected value of X, then interpret.
Game of Chance 2. (A roulette wheel)
A roulette wheel in a fiesta carnival has the numbers 1 through 30. If you bet ₱5.00 you will
have a chance to win a prepaid load worth ₱100.00. Find the expectation if you play a bet.
Game of Chance 3. (Lottery)
A lotto works by picking 6 numbers from 1- 42 (Combinations of numbers from 1-42 taken 6
at a time produce 5,245,786 number combinations). A ticket costs ₱24.00 to play the lottery. If
you win today, you would win 5 million pesos after taxes. If you play the lottery today, what would
be your expected winnings or losses?
Take It or Leave It 4. (Body Parts Insurance Investment)
An insurance company will sell a ₱500,000.00 one-year term legs insurance policy for ramp
models for a premium of ₱500.00. Find the expected value to the company of a single policy if a
model has a 99.96% chance of being uninjured in one year?
POST-TEST
You are now ready to take another chance to know where you are so far. With your effort
and determination I believe you can do it once again. Good Luck.
Direction: Solve the following real life problems.Write your answer in activity/assessment
notebook.
1. One thousand tickets are sold for ₱50.00 each.One ticket will win ₱10,000.00, two tickets will
win ₱5,000.00 each and three tickets will win ₱1,000.00 each. If X denotes the net gain from the
purchase of a randomly selected ticket, which of the following tables of values shows the
probability distribution of X?
x 10,00 5,000 1,000 -50
A. 0
P(x) 1 2 3 994
1000 1000 1000 1000
x 10,00 5,000 1,000 -50
B. 0
P(x) 1 2 3 1
999 998 997 999
x 9,950 4,950 950 -50
C. P(x) 1 2 3 994
1000 1000 1000 1000

x 9.950 4,950 1,000 -50


D. P(x) 1 2 3 1
999 998 997 999

2. What is the mean or the expected value of item number 1?(Refer to problem number 1)
A. - 25 B. - 27 C. 25 D. 27
3. You buy three ₱1,000.00 raffle tickets for a prize of a new 20- passenger Sarao jeepney valued
at ₱800, 000.00. Two thousand tickets are sold. What is the probability of winning the prize in the
purchase of three tickets?
1 2 3 4
A. B. C. D.
2000 2000 2000 2000
4. A roulette wheel in an amusement park has the numbers1 through 60. If you bet ₱100.00 you
will have a chance to win a cellular phone worth 5,000.00. How much the organizer will earn if 100
games will be played?
A. ₱50,000.00 C. ₱150,000.00
B. ₱100, 000.00 D. ₱200,000.00
5. A life insurance company will sell a 1 million three-year term life insurance policy exclusive in a
particular risk group for a premium of ₱2,000.00. Find the expected value to the company of a
single policy if a person in this risk group has a 99.95% chance of surviving three years?
A. ₱1,500.00 C. ₱1,800.00
B. ₱1,690.00 D. ₱1,950.00

CHAPTER II: NORMAL DISTRIBUTION


LESSON 1
Understanding z – scores

INTRODUCTION
This lesson deals with identifying regions under the normal curve corresponding to
different standard normal values. It also includes steps/directions on how to use the table of areas
under the normal curve. We can also apply the competency on how to construct a normal curve as
a springboard to the lesson.
This lesson also unveils the steps on how to find the area that corresponds to z –value.
Knowledge and skills in performing addition and subtraction of decimals are prerequisite of the
lesson.

OBJECTIVES/COMPETENCIES – ESTIMATED TIME


Identifies regions under the normal curve 1 day
corresponding to different standard normal values.

PRE-ASSESSMENT
Activity 1– PRE-ASSESSMENT1
Directions:Choose the letter that corresponds to the BEST answer.Write your answer in your
activity/assessment notebook.
1. What is the sum of the area that corresponds to the right of 0 and to
the left of 0?
A. 0.5 B. 1 C. 1.5 D. 2

2. Where can we find the area of specific region under the curve?
A. Table of Areas of Geometric Figure C. Table of Contents
B. Table of Areas Under Normal Curve D. Periodic Table

3. What is the area between z = - 1.23 and z = 2?


A. 0.0865 B. 0.4772 C. 0.8679 D. 0.8779

4. Find the area of the shaded region of the given figure.

A. 0.0865 B. 0.4772 C. 0.3907 D. 0.8413


5. Which of the following does NOT belong to the group?
A .to the right B. less than C. to the left D. below

MOTIVATION
Activity 2– Identify Me!
A. Direction: The following numbers are the number found at the left side and upper part of the
table. Identify the number that lies on the intersection between the given set of numbers.Write your
answerin your activity/assessment notebook. Label your work as Q3-Chapter 2 –Lesson 1-
a. 2.5 and 0.02 = _____
b. 1.0 and 0.07 = _____
c. 1.1 and 0.06 = _____
d. 0.5 and 0.08 = _____
e. 1.0 and 0.00 = _____
B. Perform the indicated operation. Copy the format and write your answer in activity/assessment
notebook.
Process Questions:
Direction: Answer the questions below. Write your answer in your activity/assessment notebook.
1. What have you noticed on the first activity?
2. How would you described the shaded part of each curve with regards to the number on the
horizontal line?
3. Refer to Activity B, what are the similarities or differences of number 1 and 2? number 3,4
and 5?
4. When do you add or subtract the obtained value from the table of the area under normal
curve?
5. Is it possible that the answer will be a negative? Why?

INSTRUCTION/DELIVERY
In the previous lesson, we discussed the properties of the normal probability distribution and
one of them that we learned is the total area under the curve is equal to one. Since the standard normal
distribution is defined as normally distributed random variable having a mean of zero ( µ= 0 ) and standard
deviation of one (ơ = 1), the fifty (50) percent of the region under the curve is below the mean and the other
50% is above the mean. The illustration is provided below.
We can easily identify the area of the regions under normal curve by using the Table of
Areas under the Normal Curve which is also known as z-Table (see attachment at the last page of
this chapter). This table gives an area to any value of z from -3.99 to 3.99. The value from this
table will describe the area of the specific region of the curve to the left of the given z-value.
How to use the Table of Areas under the Normal Curve?
Example: Find the area to the left of -1.69
Solution: First, split the given z-value into hundredths, we can find the whole number and the
tenths digit (-1.6) at left side of the table while the hundredths (0.09) located at the upper most of
the table. Then the intersection of these numbers will be the area of the normal curve to the left of
the z-value.
Illustration:

Answer: The area to the left of z= -1.69 is 0.0455.


STEPS ON HOW TO FIND THE AREA THAT CORRESPONDS TO Z –VALUE
1. Draw/sketch a normal curve and locate the given z-value on the normal curve
2. Shade the region of the curve according to the condition of z-value whether it is below,
above or between.
3. Use the table of the area under the normal curve to find the corresponding area
4. Choose the appropriate operation based on step 2 and 3
4.1 When the z-value is to the left or any related terms (e.g. below, less than) just write the
value we obtained in step 3
4.2 When the z-value is to the right or any related terms (e.g. above, greater than), subtract
1 by the obtained value in step 3
4.3 When the shaded region is in between of the two z-value, subtract the biggest by
smallest value obtained in step 3
5. Label the shaded region.
Example 1: Find the area that corresponds below z = -1.35
Solution:
Step 1. Draw/sketch a normal curve and locate the given z-value.

Step 2: Shade the region of the curve according to the condition of z-value whether it
is below, above or between.

Step 3. Use the table of the area under the normal curve to find the corresponding
area

The intersection between -1.3 and 0.05 is 0.0885

Step 4. Choose the appropriate operation based on step 2 and 3


When the z-value is to the left or any related terms (e.g. below, less than) just write the value we
obtained in step 3.
Since the shaded region of the curve is to the left of the z= -1.35 and the intersection between -1.3
and 0.05 is 0.0885. Therefore, the area of the shaded region is 0.0885.
Step 5. Label the shaded region.
Example 2: Find the area to the right of z = -1.35
Step 1:

Step 2:

Step 3.
The intersection between -1.3 and 0.05 is 0.0885

Step 4.
Since the shaded region is to the right of z-value, we will subtract 0.0885 to 1.
The difference is 0.9915
Step 5

PRACTICE
Direction: Find the area that corresponds to each of the following z values. Sketch and label each
curve. Write your answer in your activity/assessment notebook.
1. to the right of z = 2.18
2. below z= -1.40
3. above z= 0.57
4. between z = -2.34 and z = 1.57
5. between z = 0.32 and z = 2.42

VALUES INTEGRATION
Direction: Please share your own thoughts about the text inside the box relating to our Core
Values NOTRE DAME. Write your answer in your activity/assessment notebook.

An approximate answer to the right problem is worth a good


deal more than an exact answer to an approximate problem.

John Tukey

POST-TEST
Directions: Choose the letter that corresponds to the BEST answer. Write your answer in your
activity/assessment notebook.

1. What will you do to find the area between the two z values?
A. Subtract the obtained value from the Z table to 1
B. Rewrite the value obtained from z table
C. Subtract the two obtained value from the z table.
D. Calculate the mean and standard deviation
2. What is the another term of z table?
A. Table of Areas of Geometric Figure C. Table of Contents
B. Table of Areas Under Normal Curve D. Periodic Table
3. What is the area between z = 0.23 and z = 1.99 ?
A. 0.3858 B. 0.3588 C. 0.8239 D. 0.8583

4. Which of the following is the best illustration of the area to the left of z =1?
A. B. C. D

5. Find the area of above z= 2.14


A. 0.052 B. 0.0162 C. 0.0324 OD. 0.9838
LESSON 2
Converting Raw Score to Standard z – score
INTRODUCTION
In this lesson, you will learn how to convert a normal random variable X to standard
normal variable z and vice-versa. Moreover, this self-learning kit in this particular lesson,
will enable you solve real-life problems.
Included in this self-learning kit are pre-test and post-test. Please answer the pre-
test and post-test in order to assess the progress of your learning.
OBJECTIVES/COMPETENCIES – ESTIMATED TIME
Converts a normal random variable to a standard
normal variable and vice versa. 2days

PRE-ASSESSMENT
Activity 1– PRE-ASSESSMENT1
Direction: Read and analyze each of the following statements carefully. Choose the letter that
corresponds to the letter of the correct answer.Write your answer in your activity/assessment
notebook.
1. If the scores in a Probability and Statistics test are normally distributed with a mean of 25 and
standard deviation of 3.5. What is the z-score for a score of 30?
A. 1.43 B. -1.43 C. 1.06 D. 0.72
2.Gracia, an ALS student, obtained a z-score of 2.35 in an achievement test that is normally
distributed with a mean grade of 83 and standard deviation of 5. What was Gracia’s grade in the
achievement test?
A. 71.25 B. 94.75 C. 85.35 D. 80.65
3.In a normal distribution with a mean of 15 and standard deviation of 3, what does a z-score of -2
indicate?
A. It indicates that the score is below the mean.
B. It indicates that the score is equal to 9.
C. It indicates that the difference between the score and the mean is 6.
D. All of the above
4. Ronnie and Loida who are best friends took a college entrance examination. The mean score
and standard deviation of the examination are 110 and 15 respectively. If Ronnie and Loida
obtained z- scores of 1.25 and -0.75 respectively, what was their scores in the examination?
A. 128.75 and 98.75
B. 120.75 and 88.75
C. 130 and 99
D. 140.75 and 100.75
5. The velocities of cars in express way are normally distributed with a mean of 90 km/hr and
standard deviation of 10 km/hr. Jessie’s velocity has a z-score of 2.5, what was his
velocity?
A. 100 km/hr B. 105 km/hr C. 110 km/hr D. 115 km/hr
MOTIVATION
Activity 2– Complete Me!
Direction: Complete the table below by filling in the correct value of the required variable. Recall
x−μ
that z = .
σ
Z x μ σ
___ 23 18 4
-1.17 ___ 125 30
___ 515 475 100
0.565 ___ 15.5 5.5

Process Questions:

This time, you need to reflect on the following questions. Answer the following
questions:
1. How did you find the activity?
2. How did you able to find the value of z? x?
3. Can you derive a formula in finding the value of x?
4. How about for the value of μand σ ?

INSTRUCTION/DELIVERY
In solving real-life problems involving the normal curve, it is very important to convert a
random variable x to a standard normal variable or z-score. This procedure is known as
“standardizing” or “standardization” of a random variable, where a standardized value is called
a z-score. A z-score is a measure of the number of standard deviations (σ ) a particular data value
is away from the mean (μ). Suppose your score on a test in Probability and Statistics was 39 and
the scores are normally distributed with a mean of 33 and standard deviation of 3, then your score
is exactly 2 standard deviations above the mean. If you scored 30, then it is exactly 1 standard
deviation below the mean. All values that are above the mean have positive z-scores and all
values that are below the mean have negative z-scores. If you obtained a z-score of -3, this
means that your score is 3 standard deviations below the mean, that is, 33 – 3(3) = 33 – 9 = 24.
Given any value x from a normal distribution with mean μ and standard deviation σ , to
convert x to a z-score (standard normal score), you need to;
Subtract the mean μ from x.
Divide this quantity, x – , by the standard deviation σ .
In equation, a z-score is defined as:
x−μ
z=
σ
where: z – standard normal score or z – score
x – any data value in a normal distribution
μ - mean
σ - standard deviation
Example 1.
Suppose IQ scores are normally distributed with a mean of 100 and standard deviation of
10. If your IQ is 85, what was your z-score?
Solutions:
The z-score can be computed using the formula;
x−μ
z=
σ
85−100 −15
z= = = -1.5
10 10
Example 2.
On a nationwide placement test that is normally distributed, the mean was 125 and standard
deviation was 15. If you scored 149, what was your z-score?
Solutions:
x−μ
Using the formula z = ,
σ
149−125
z=
15
24
=
15
= 1.60

Example 3.
The heights of teachers in Sta. Catalina National High School are normally distributed with a mean
of 150 cm and standard deviation of 15 cm. The height of Sir Victor has a z-score of 3.25. What is
the actual height of Sir Victor?

Solutions:
In this example, z-score is given while the normal random variable is unknown. To solve for
the normal random variable x, multiply the z-score (z) by the standard deviation σ , then add the
mean μ. Therefore;
x = zσ + μ
= 3.25(15 cm) + 150 cm
= 48.75 cm + 150 cm
= 198.75 cm

PRACTICE
Direction: Solve the following problems. Write your answer in your activity/assessment notebook.
1. The mean number of hours a Filipino worker spends on the computer is 3.1 hours per
workday. Assume the standard deviation is 0.5 hour and is normally distributed, how long
does a worker spend on the computer if his z-score is 1.25?
2. Each month, a Filipino household generates an average of 28 pounds of newspaper for
garbage or recycling. Assume the standard deviation is 2 pounds. Determine the z-score of
a household that generates 22 pounds of newspaper.
3. The Candelaria Automobile Association reports that the average time it takes to respond to
an emergency call is 30 minutes. Assume the variable is normally distributed and the
standard deviation is 4.5 minutes. How long will a call be responded if it has a z-score of
0.75?
4. The average monthly salary for first-year teachers is P21,945. If the distribution is
approximately normal with a standard deviation of P3250. How much will a teacher earn in
a month if his salary has a z-score of 1.15?
VALUES INTEGRATION
Direction: Please share your thoughts about the saying inside the box. Write your answer in your
activity/assessment notebook.

“Given a large mass of data, we can by judicious selection construct perfectly


plausible unassailable theories – all of which, some of which, or none of
which may be right” (Paul Arnold Srere)

POST TEST
Direction: Read and analyze each of the following statements carefully. Then shade the circle
that corresponds to the letter of the correct answer.
1. If the scores in a Probability and Statistics test are normally distributed with a mean of 25 and
standard deviation of 3.5. What is the z-score for a score of 30?
A. 1.43 B. -1.43 C. 1.06 D. 0.72
2. Gracia, an ALS student, obtained a z-score of 2.35 in an achievement test that is normally
distributed with a mean grade of 83 and standard deviation of 5. What was Gracia’s
grade in the achievement test?
A. 71.25 B. 94.75 C. 85.35 D. 80.65
3. In a normal distribution with a mean of 15 and standard deviation of 3, what does a z-
score of -2 indicate?
A. It indicates that the score is below the mean.
B. It indicates that the score is equal to 9.
C. It indicates that the difference between the score and the mean is 6.
D. All of the above
4.Ronnie and Loida who are best friends took a college entrance examination. The mean
score and standard deviation of the examination are 110 and 15 respectively. If Ronnie
and Loida obtained z-scores of 1.25 and -0.75 respectively, what was their scores in the
examination?
A. 128.75 and 98.75
B. 120.75 and 88.75
C. 130 and 99
D. 140.75 and 100.75
5. The velocities of cars in express way are normally distributed with a mean of 90 km/hr
and standard deviation of 10 km/hr. Jessie’s velocity has a z-score of 2.5, what
was his velocity?
A. 100 km/hr B. 105 km/hr C. 110 km/hr D. 115 km/hr

LESSON 3
Identifying Regions of Areas Under the Normal Curve
INTRODUCTION
In this lesson, you will learn how to apply the z-score in finding probabilities and percentiles by
simply looking at the Table of Standard Normal Distribution. Moreover, this self-learning kit in this
particular lesson, will enable you to solve real-life problems about probabilities and percentiles.
OBJECTIVES/COMPETENCIES – ESTIMATED TIME
Computes probabilities and percentiles using the
standard normal distribution table. 2days
PRE-ASSESSMENT
Activity 1– PRE-ASSESSMENT1
Direction: Read and analyze each of the following statements carefully. Then shade
the circle that corresponds to the letter of the correct answer.
1. Which of the following notations indicate the probability of a z value from the left?
A. P(Z > z) B. P(Z < z) C. P(X < x) D. P(X > x)
2. What is the probability of z value indicated by P(Z < -1.78)?
A. 0.0375 B. 0.9625 C. 0.9633 D. 0.3075
3. The ages of ALS students enrolled at Sta. Catalina National High School are normally
distributed with a mean of 19 years and standard deviation of 3.5 years. If a student is selected
at random, what is the probability that his age is under 22 years?
A. 19.49% B. 80.51% C. 11.67% D. 90.57%
4. The weekly sales of ABM students on their “Go Negosyo” project are normally distributed with
mean of P3,200 and standard deviation of P500. What percent of their weekly sales are over
P4,000?
A. 94.52% B. 77.34% C. 5.48% D. 22.57%
5. To lessen the number of non-numerates in Sta. Catalina National High School, the Math
department implemented Project MEAN (Masterful Elimination of All Non-numerates). A
student is required to undergo with the program if he/she scored below 20 in the numeracy
test. If the scores of the students in the numeracy test are normally distributed with a mean of
30.50 and standard deviation of 9.5, what percentage of the students needed to undergo with
the program?
A. 13.35% B. 86.65% C. 11.27% D. 14.45%
MOTIVATION
Activity 2– Survey Me!
Direction: Find the area of the shaded region under the normal curve.Write your answer in your
activity/assessment notebook
1. 2.

3. 4.

5. 6.
Process Questions: Answer the the questions below. Write your answer in your
activity/assessment notebook.
1. How did you find the activity?
2. How did you find the area of the shaded region at the left of a given z-score?
3. How did you find the area of the shaded region with a negative z-score? positive z-score?
4. How did you find the area of the shaded region in between two given z-scores?
5. How did you find the area of the shaded region at the right of a given z-score?
6. How did you find the area of the shaded region at the left of a given normal random variable
X?
7. How did you find the area of the shaded region in between two given normal random
variables?

INSTRUCTION/DELIVERY
A normal distribution curve can be used as a probability distribution curve for normally
distributed variables. The area under the standard normal distribution curve can also be thought of
as a probability. That is, if it were possible to select any z value at random, the probability of
choosing one, say, at the left of 1.45 would be the same as the area under the curve at the left of
1.45. In this case, the area is 0.9265. Therefore, the probability of randomly selecting a z value at
the left of 1.45 is 0.9265 or 92.65%. The problems involving probabilities and percentiles are
solved in the same manner as finding the areas under a normal curve.
In finding probabilities, the following notations will be used;
a. P(Z¿z) – Probability at the left of z.
b. P(Z ¿ z) = 1 – P(Z ¿ z) – Probability at the right of z
c. P(a ¿z ¿ b) – Probability of z that is in between two other z values a and b.
d. P(X¿ x) – Probability at the left of a normal random variable x
e. P(X ¿ x) – Probability at the right of a normal random variable x
f. P(a ¿ X ¿ b) – Probability of a normal random variable X that is in between two other
normal random variables a and b.
Example 1.
Find the probabilities for each of the following;
(a) P(Z ¿ 1.32)
(b) P(Z ¿ -1.05)
(c) P(-0.75 ¿ Z ¿ 1.56)
(d) P(Z ¿ -0.88

Solutions:
The probability P(Z ¿ 1.32) means to find the area under the normal curve at the left of z = 1.32.
From the table, find the intersection of the row 1.3 and the column 0.02. Thus, the probability is
0.9066 or 90.66%.
a. The probability P(Z ¿ -1.05) is the same as the area under the normal curve at the left of z =
-1.05. From the table, the area is 0.1469. Thus, the probability is 0.1469 or 14.69%.

b. The probability P(-0.75 ¿ Z ¿ 1.56) is the same as the area under the normal curve between z
values -0.75 and 1.56. To get the area, subtract the area of z = -0.75 from the area of z = 1.56.
Thus, P(-0.75 ¿ Z ¿ 1.56) = P(Z ¿1.56) – P(Z ¿ -0.75) = 0.9406 – 0.2266 = 0.7140.

c. The probability P(Z ¿ -0.88) is the same as the area under the normal curve at the right of z =
-0.88. To get the area, subtract the area of z = -0.88 from 1. Thus, P(Z ¿ -0.88) = 1 – P(Z ¿ -0.88) =
1 – 0.1894 = 0.8106.
PRACTICE

This time, it’s your turn to shine. Answer the following problems.
Direction: Solve the following problems. Write your answer in your activity/assessment notebook.
1. Use the Table of Standard Normal distribution to find the probabilities of the following;
a. P(Z ¿-1.45)
b. P(0.5 ¿ Z ¿ 2.33)
c. P(Z ¿ 1.78)
2. X is a normally distributed random variable with a mean of 60 and standard deviation of 8. Find
the probabilities indicated by using the table.
a. P(X ¿ 52)
b. P(48 ¿ X ¿ 64)
c. P(X ¿ 57)
3. The average number of calories in a 1.8-ounce chocolate bar is 230. Suppose that the
distribution of calories is approximately normal with a standard deviation of 10. Find the probability
that a randomly selected chocolate bar will have less than 200 calories.

4. The average monthly salary of 500 teachers of Candelaria East district is P26,500. If the
monthly salaries of the teachers are normally distributed with standard deviation of P2,500,
approximately how many teachers in the district are earning more than P30,000 monthly?
VALUES INTEGRATION
Direction: Write your reflection about the saying inside the box. Write your answer in your
activity/assessment notebook.

For the most part, statistics is a method of investigation that is used when
other methods are of no avail; it is often a last resort and a forlorn hope.
Michael J. Moroney

POST-TEST
Direction: Read and analyze each of the following statements carefully. Then shade the circle
that corresponds to the letter of the correct answer.

1. Which of the following notations indicate the probability of a z value from the left?
A. P(Z > z) B. P(Z < z) C. P(X < x) D. P(X > x)
2. What is the probability of z value indicated by P(Z < -1.78)?
A. 0.0375 B. 0.9625 C. 0.9633 D. 0.3075
3. The ages of ALS students enrolled at Sta. Catalina National High School are normally
distributed with a mean of 19 years and standard deviation of 3.5 years. If a student is selected
at random, what is the probability that his age is under 22 years?
A. 19.49% B. 80.51% C. 11.67% D. 90.57%
4. The weekly sales of ABM students on their “Go Negosyo” project are normally distributed with
mean of P3,200 and standard deviation of P500. What percent of their weekly sales are over
P4,000?
A. 94.52% B. 77.34% C. 5.48% D. 22.57%
5. To lessen the number of non-numerates in Sta. Catalina National High School, the Math
department implemented Project MEAN (Masterful Elimination of All Non-numerates). A student
is required to undergo with the program if he/she scored below 20 in the numeracy test. If
the scores of the students in the numeracy test are normally distributed with a mean of 30.50
and standard deviation of 9.5, what percentage of the students needed to undergo with the
program?
A. 13.35% B. 86.65% C. 11.27% D. 14.45%
CHAPTER III: SAMPLING AND SAMPLING DISTRIBUTION
LESSON 1
Sampling Distribution of Sample Means

INTRODUCTION
In Statistics, it is sometimes impossible to gather data from the entire population or
collection of objects being examined in the study but through the use of samples or
representatives of the population, researchers get a clearer picture of the whole group being
studied. The method of getting samples is called sampling where it needs not to be biased to give
justice to the group it represents. Random sampling is a method wherein each of the members of
the population has a chance of being taken as sample.
OBJECTIVES/COMPETENCIES – ESTIMATED TIME
Illustrates a random sampling.
Distinguish between parameter and statistic. 5 days
Identifies sampling distributions of statistics (sample
mean).

PRE-ASSESSMENT

Activity 1– PRE-ASSESSMENT1
Direction: Read and understand the questions below. Choose the letter of the correct
answer and write it in your activity/assessment notebook.
The Philippine High School has 5,000 senior students. Mr. Manalo, the principal, wants to obtain
information from the senior high students as to their plans after graduation.
1. Principal Manalo writes all the names of the senior high school students on small pieces of paper
and draws 500 names to participate in the study. Which random sampling technique does he apply?
A. cluster B. simple C. stratified D. systematic
2. The principal creates a list of all senior high students, decides to survey every tenth name on the list
and later asks those students that are selected. Which random sampling technique does he apply?
A. cluster B. simple C. stratified D. systematic
3. Mr. Manalo divides the senior high students as to their grade level and strand. He proportionately
chooses students from each strand. Which random sampling technique does he apply?
A. cluster B. simple C. stratified D. systematic
4. The principal divides the senior high school students depending on the barangay where they live and
randomly picks some of the barangays to answer the questionnaire. Which random sampling technique
does he apply?
A. cluster B. simple C. stratified D. systematic
5. All LRN of each senior high student are written in pieces of paper and placed in a bowl. Mr. Manalo
picks the samples based from the table of numbers. Which random sampling technique does he apply?
A. cluster B. simple C. stratified D. systematic

MOTIVATION
The key to understand random sampling is to understand first the different types of it and know the
reason why those kinds of sampling becomes random.
Activity 2– Match Me!
A. Direction: Match the word from column A to its definition in column B. Write your answer in
your activity/assessment notebook.
Column A Column B
____ 1. Simple a. methodical procedure
____ 2. Systematic b. to divide or arrange into classes
____ 3. Stratified c. a number of similar things that occur together
____ 4. Cluster d. having few parts; not complex

Since you are already familiar with random sampling words, identify the random sampling
used in each of the following situations.
B. Direction: Choose from simple, systematic, stratified and cluster.
______________ 1. A researcher writes the name of each student on a piece of paper, mixes the
papers in a bowl, and draws 7 pieces of paper.
______________ 2. A researcher selects every 7th students from a list.

______________ 3. A researcher tells the class to count off, and then selects those numbers that
are multiple of 7.
______________ 4. A researcher separates the list from boys and girls then draws 7 names per
each gender.
______________ 5. A researcher surveys all students from 3 randomly selected classes out of 7
classes.
Process Questions:
Direction: Challenge yourself to think about the questions given based from our activity. Write
your answer in your activity/assessment notebook.

1. What is a random sampling?


2. What are the different types of random sampling?
3. How do they differ from one another?
INSTRUCTION/DELIVERY
Let us analyze the situations given earlier.
1. A researcher writes the name of each student on a piece of paper, mixes the papers in a bowl,
and draws 7 pieces of paper.
Situation number 1 uses simple random sampling since the pieces of paper represent the
students as elements of the population. All of them have an equal chance of being selected as
samples and the method of selection involves picking those 7 pieces of paper as samples.

2. A researcher selects every 7th students from a list.


3. A researcher tells the class to count off, and then selects those numbers that are multiple of 7.
Situations number 2 and 3 apply systematic random sampling since samples are being
selected based from the kth consistent intervals. On these cases, 7 th names on the list and
multiple of 7 were the interval used.
4. A researcher separates the list from boys and girls then draws 7 names per each gender.
Situation number 4 uses stratified random sampling because the students are divided into
two different strata, boys and girls and then the selection of samples in each gender was done.
5. A researcher surveys all students from 3 randomly selected classes out of 7 classes.
Situation number 5 practices cluster sampling since all students are divided into clusters or
classes and then the classes were randomly selected. All of the students in the classes that were
randomly selected are considered the samples of the study.
Random Sampling is a sampling method of choosing representatives from the population
wherein every sample has an equal chance of being selected.
There are different types of random sampling.
a. Simple random sampling is the most basic random sampling wherein elements are usually
represented by a number that are written on equal sized and shaped papers and then selection of
samples is possible through the lottery method. Drawing number of paper is based from the
desired number of sample.
b. Systematic random sampling is a random sampling which uses a list of all the elements in the
population and then elements are being selected based from the kth consistent intervals. To get
the kth interval, divide the population size by the sample size.
c. Stratified random sampling is a random sampling wherein the population is divided into different
strata. Selection of samples will be proportionately randomly picked in each stratum that is
why all strata have representatives in forming the samples.
d. Cluster sampling is a random sampling wherein population is divided into clusters and then the
clusters is randomly selected. All elements of the clusters randomly selected are considered
the samples of the study.

PRACTICE
Since you already know a lot of things about random sampling, let us assess that knowledge!

A. Direction: Identify the random sampling method used in each item. Write your answer in your
activity/assessment notebook.
_______________1. You are given a list of all graduating students in your school. You decide to
survey every tenth name on the list and ask in which organization they belong.
_______________2. You wish to make a comparison about the gender differences on
Mathematics performance. You divide the population into two groups, male and female, and
randomly pick respondents from each of the gender.
_______________3. You assign numbers to the members of the population and then use draw
lots to obtain your samples.
_______________4. You randomly pick five out of fifteen barangays to where to conduct your
survey in your municipality or city.
_______________5. You write the names of each student in pieces of paper, shuffles and then
draw eight names.
B. Direction:Each of the 30 basketball high school teams has 12 players. The organizer wants to have a
quick survey to know the average height of the players. Write your answer in your activity/assessment
notebook.
1. Each team will be asked to place papers with its players’ names into a fishbowl and
randomly drew out five names. The five names from each team will be combined to make up
the sample. Which of the following sampling techniques is being used in this situation?
A. systematic B. stratified C. simple D. cluster
2. The organizer will create a list of all players and they will be represented by a number. Sixty
numbers will be picked to know the samples. Which random sampling technique did he apply?
A. systematic B. stratified C. simple D. cluster
3. All players will be grouped according to their age and will proportionately choose players to
measure their height. Which random sampling technique did he apply?
A. systematic B. stratified C. simple D. cluster
4. The organizer created a list of all players, decided to survey every sixth name on the list and
later asked those players that were selected. Which random sampling technique did he apply?
A. systematic B. stratified C. simple D. cluster

LESSON 2
Sampling Distribution of Means

INTRODUCTION
If the number of elements of the population is too many, it will be extremely hard for the
researchers to collect all data from the group. In some cases, we need to use random sampling to
get appropriate statistic from the samples to draw conclusion that would represent the population
but there can be numerous different set of possible samples of the same size that can be drawn
from a given population. Sampling distribution shows all the possible samples together with their
mean. It describes the probability for each mean of all samples with the same sample size n.
OBJECTIVES/COMPETENCIES – ESTIMATED TIME
Finds the mean and variance of the sampling 2 days
distribution
of the sample mean

PRE-ASSESSMENT
Activity 1– PRE-ASSESSMENT1
Direction: Read and understand each statement below. Choose the letter of the correct answer. Write your
answer in your activity/assessment notebook. Label your work as Q3-Lesson 2- Activity 1-Pre-assessment.
For numbers 1-5, a population consists of the data (1, 2, 3, 4).

1. How many different samples of size n = 2 can be drawn from the population?
A. 6 B. 5 C. 4 D. 3
2. Which of the following sample means is the most frequent in the sampling distribution of sample
means?
A. 1.5 B. 2 C. 2.5 D. 3
3. What is the probability of the sample mean 3.5 in the sampling distribution of sample means?
2 1 3 2
A. B. C. D.
3 6 4 6
4. What is the lowest value of the sample mean in this sampling distribution?
A. 0 B. 1.5 C. 2 D. 2.5
5. Which of the following histogram correctly shows the sampling distribution of sample means of size n =
2?
A. C.

0 1.5 2 2.5 3 3.5 0 1.5 2 2.5 3 3.5


B. D.

0 2 2.5 3 3.5 4 0 2 2.5 3 3.5 4


MOTIVATION
In this lesson, you will learn to construct a sampling distribution of the sample means as well as its
histogram. Answer this activity to continue learning the concept.
Activity 2– Evaluate Me!
Direction: Evaluate the following combination of n objects taken at a time. Write your answer in
your activity/assessment notebook. Label your work as Q3-Lesson 2-Activity 2-Evaluate Me!
__________ 1. 4C2
__________ 2. 6C2
__________ 3. 5C3
A population consists of the numbers 1, 2, 3, 4 and 5. Let us list all possible samples of size
3 from this population and compute the mean of each sample. Complete the table. Copy the
format and write your answer in your activity/assessment notebook.
Observation Sample Mean
1 1, 2, 3 (1 + 2+ 3) ÷ 3 = 6 ÷ 3 = 2
2 1, 2, 4 2.33
3 1, 2, 5
4 1, 3, 4
5 1, 3, 5
6 1, 4, 5
7 2, 3, 4
8 2, 3, 5
9 2, 4, 5
10 3, 4, 5
There are 10 possible samples of size 3 that can be drawn from the given population.
To get this, we apply the formula for combination of N taken n at a time NCnwhere N isthe size of
the population and n is the sample size.
Then, let us make a frequency distribution of the sample means. This is now the sampling
distribution of the sample means.
Sample Mean Frequency
2 1
2.33 1
2.67 2
3 2
3.33 2
3.67 1
4 1
Challenge yourself to think about the questions given based from our activity.
Process Questions: Write your answer in your activity/assessment notebook.

1. What is sampling distribution of sample means?


2. How are we going to create a sampling distribution of sample means? What are the steps?
3. How are we going to create a histogram of the sampling distribution of sample means?
INSTRUCTION/DELIVERY
A sampling distribution of the sample means is the frequency distribution table using the
computed sample means from all the possible random samples of a particular sample size taken
from the given population.

Phases to make the sampling distribution of the sample means:


1. Determine the number of set of all possible random samples that can be drawn from the given
population by using the formula NCnwhere N is thepopulation size and n is the sample size.
2. List all the possible random samples and solve for the sample mean of each set of samples.
3. Construct a frequency distribution table of the sample means indicating its number of
occurrence or the frequency and together with its probability.

Since you already know how to construct sampling distribution and histogram, let
us apply your knowledge in the following data below.

PRACTICE
I. A population consists of the values (1, 4, 3, 2). Consider samples of size 2 that can be drawn
from this population.
a. List down all the possible samples and corresponding sample mean. Write your answer in your
activity/assessment notebook.
Observation Samples Sample Mean
1
2
3
4
5
6
b. Construct the sampling distribution of the sample means. Copy the format and write your
answer in your activity/assessment notebook.
Observation Sample Mean Frequency Probability P(x)
1
2
3
4
5
6
c. Draw a histogram of the sampling distribution of the means. Copy the format and write your
answer in your activity/assessment notebook.

LESSON 3
Finding the Mean and Variance of the Sampling Distribution of Means

Statisticians do not just describe the variation of the individual data values about the mean
of the population. They are also interested to know how the means of the samples of the same
size taken from the same population vary about the population mean. In this lesson, you will learn
how to describe the sampling distribution of the sample means by computing its means and
variance.

OBJECTIVES/COMPETENCIES – ESTIMATED TIME


Finds the mean and variance of the sampling 2 days
distribution
of the sample mean

PRE-ASSESSMENT

Activity 1– PRE-ASSESSMENT1
Direction: Read and understand each question below. Choose the letter of the correct answer
and write it in your activity/assessment notebook.

A population consists of the five (5) measurements 2, 3, 6, 5, and 7.


1. What is the mean?
A. 3.44 B. 4. 60 C. 5.20 D. 4.92
2. What is the variance of the population?
A. 3.44 B. 4.60 C. 5.20 D. 4.92
3. How many different samples of size n=2 can be drawn from the population?
A. 7 B. 8 C. 9 D. 10
4. What is the mean x of the sampling distribution of the means?
A. 3.44 B. 5.20 C. 4.60 D. 4.92
5. What is the variance of the sampling distribution?
A. 3.90 B. 0.89 C. 2.20 D. 1.29

MOTIVATION
“Can you identify the steps that include mean and variance of sampling distribution of sample means?”
Study the following problem to perform this task.

Activity 2– Answer Me!


Direction: Consider a population consisting of 1, 2, 3, 4, and 5. Suppose samples of size 2 are
drawn from this population. Describe the sampling distribution of the sample means. Write your
answer in your activity/assessment notebook. Label your work as Q3- Lesson 3 –Activity 2-Answer
Me!
1. What is the mean and variance of the sampling distribution of the sample means?
2. Compare these values to the mean and variance of the population.

Illustration Steps
1.
Σx 1+ 2+ 3+4 +5
µ= = = 3.00
N 5

x x-µ (x-µ)2 2.
1 -2 4 2 Σ( x−µ)2
σ =
2 -1 1 N
3 0 0 10
4 1 1 = 5
5 2 4
Σ(x- µ)2=10 =2

So, the variance of the population is 2.


Use the formula NCn. Here N = 5 and n = 2. 3.
5C2 = 10
So there are 10 possible samples of size 2 that can be drawn.
Samples Mean 4.
1,2 1.5
1,3 2.0
1,4 2.5
1,5 3.0
2,3 2.5
2,4 3.0
2,5 3.5
3,4 3.5
3,5 4.0
4,5 4.5

Sampling Distribution of Sample Means 5.


Sample Frequency Probability
Mean P

1.5 1 1/10
2.0 1 1/10
2.5 2 1/5
3.0 2 1/5
3.5 2 1/5
4.0 1 1/10
4.5 1 1/10
Total 10 1.00

Probability 6.
Sample Mean P • P

1.5 1/10 0.15


2.0 1/10 0.20
2.5 1/5 0.50
3.0 1/5 0.60
3.5 1/5 0.70
4.0 1/10 0.40
4.5 1/10 0.45
Total 1.00 3.00

µx = Σ • P
= 3.00

Sample Probabilit Mean-µ 7.


2 2
Mean y (Mean-µ) Probability•((Mean-µ)

P
1.5 1/10 -1.5 2.25 0.225
2.0 1/10 -1.0 1.00 0.100
2.5 1/5 -0.5 0.25 0.050
3.0 1/5 0.0 0.00 0.000
3.5 1/5 0.25 0.25 0.050
4.0 1/10 1.00 1.00 0.100
4.5 1/10 2.25 2.25 0.225
Total 1.00 0.750
So, the variance of the sampling distribution is 0.75

INSTRUCTION/DELIVERY

Here are the basic steps in calculating mean and variance of the sampling distribution of means.

Illustration Steps
1. Compute the
Σx 1+ 2+ 3+4 +5 mean of the
µ= = = 3.00
N 5 population.

x x-µ (x-µ)2 2. Compute the


1 -2 4 variance of the
2 -1 1 population.
3 0 0
4 1 1
5 2 4
Σ(x- µ)2=10

σ2 = Σ(x- µ)2/ N
= 10 / 5
=2

So, the variance of the population is 2.

Use the formula NCn. Here N=5 and n=2. 3. Determine the
number of possible
5 C2 = 10 samples of size n =
2.
So there are 10 possible samples of size 2 that can be drawn.

Samples Mean 4. List all possible


1,2 1.5 samples and their
1,3 2.0 corresponding
1,4 2.5 means.
1,5 3.0
2,3 2.5
2,4 3.0
2,5 3.5
3,4 3.5
3,5 4.0
4,5 4.5

Sampling Distribution of Sample Means 5. Construct the


Sample Frequency Probability sampling
Mean distribution of the
P sample means.

1.5 1 1/10
2.0 1 1/10
2.5 2 1/5
3.0 2 1/5
3.5 2 1/5
4.0 1 1/10
4.5 1 1/10
Total 10 1.00

Sample Probability
Mean P P 6. Compute the
mean of the
sampling
1.5 1/10 0.15 distribution of the
2.0 1/10 0.20 sample means.
2.5 1/5 0.50 Follow these steps:
3.0 1/5 0.60 a. Multiply the
3.5 1/5 0.70 sample mean by
4.0
4.5 µx = Σ1/10

1/10 P
0.40
0.45
the corresponding
probability.
Total 1.00 3.00 b. Add the results.
= 3.00

Sample Probability Mean- 7. Compute the


Mean µ (Mean-µ) 2
Probability•((Mean- variance of the
P µ)2 sampling
distribution of the
1.5 1/10 -1.5 2.25 0.225 sample means.
2.0 1/10 -1.0 1.00 0.100 Follow these steps:
2.5 1/5 -0.5 0.25 0.050 a. Subtract the
3.0 1/5 0.0 0.00 0.000 population mean
3.5 1/5 0.25 0.25 0.050 from each sample
4.0 1/10 1.00 1.00 0.100 mean.
4.5 1/10 2.25 2.25 0.225
Total 1.00 0.750 b. Square the
difference.
So, the variance of the sampling distribution is 0.75
c. Multiply the
results by the
corresponding
probability.

d. Add the results.


Based from the previous activity, the mean of the population is equal to the sample means. This
happens because all samples obtained of n size came from the same population. The variance of the
population is greater than the variance of the sample means, since a sample of n is less than the population N
then the data vary by a small amount as compared to population.
The standard error of the mean is the standard deviation of the sampling distribution of
the mean. ... Given a population with a finite mean μ and a finite non-zero variance σ2, the sampling
distribution of the mean approaches a normal distribution with a mean of μ and a variance of σ2/N as N,
the sample size, increases.
PRACTICE
Direction: Answer each question. Write your answer in your activity notebook.
Consider all samples of size 5 from this population:
2, 5, 6, 8, 9
1. Calculate the mean and standard deviation of the population.
2. List all samples of size 3 and compute the mean for each sample.
Sample Mean

3. Construct the sampling distribution of the sample means.


Sampling Distribution of Sample Means
Sample Mean Frequency Probability

Reflection

“The VARIANCE of people depends on how MEAN they can be.” -KirtiChowdhary

Direction: Write your reflection about the statement inside the box. Write your answer in your
activity/assessment notebook.

LESSON 4
Central Limit Theorem

INTRODUCTION
Many common statistical procedures require data to be approximately normal
to roughly follow the bell curve. But what happens when you have a population that is just not
normal?

OBJECTIVES/COMPETENCIES – ESTIMATED TIME


Illustrates the Central Limit Theorem.
Defines the sampling distribution of the sample 4 days
mean using the Central Limit Theorem.

PRE-ASSESSMENT
Activity 1– PRE-ASSESSMENT1
Direction: Answer the following questions by choosing the letter of the correct answer from the
choices below. Write your answer in your activity/assessment notebook.Label your work as Q3-
1. If a population has a mean of 12.8, what is the mean of the sampling distribution?
A. Less than 12.8 C. closer to 12.8
B. Larger than 12.8 D. Exactly the same as 12.8
2. If the mean of the sampling distribution of the means is 6.5, which of the following best
describes the population mean?
A. The population is greater than 6.5
B. The population decreases by 6.5
C. The population mean is also equal to 6.5.
D. The population mean and mean of the sampling distribution of the means cannot be
compared
3. Which of the following statements is NOT true about Central Limit Theorem?
A. The population mean and the mean of the sampling distribution of the means are
equal.
B. If you take repeatedly independent random samples of size n from any population,
then when n is large,
the distribution of the sample means will approach a normal distribution.
C. The central limit theorem tells us exactly what the shape of the distribution of the
means will be when we
draw repeated samples from a given population.
D. The mean of the sampling distributions of the means, standard deviation of the
sampling distribution of
the means and variance is the same as the population mean, variance of the
population and standard
deviation.
4. Consider the population consisting of values (2, 4, 6). List all the possible samples of size 2
which can be drawn
with replacement.
A. {(2,2),(2,4),(2,6),(4,2),(4,4),(4,6),(6,2),(6,4),(6,6)}
B. {(2,2),(2,4),(2,6) ,(4,2),(4,4),(4,6)}
C. {(4,2),(4,4),(4,6),(6,2),(6,4),(6,6)
D. {2,4,6}
5. If the population has a variance of 4.6, what is the variance of the sampling distribution of its
means if the sampling distribution was derived with sample size n = 2 and all possible samples
are drawn with replacements?
A. 19.22 B. 21.16 C. 10.58 D. 2.3

MOTIVATION

Activity 2– Word Puzzle


WORD PUZZLE:Direction: Circle the word or group of words that look familiar to you.
The words can be read forward, backward upward, downward or diagonally. Copy the format and
write your answer in your activity/assessment notebook. Label your work as Q3-Lesson 4-Activity
2- Word Puzzle.
Normal Distribution Population Central Limit Theorem
Standard Deviation Mean Table
Variance Sample Size Bar Graph
D E V I A T I O N C
G C Z T D E F C A E
R I A I J B A R E N
A D N O S E P R M T

P S A M P L E V M R

H N O R M A L E U A

P O P U L A T I O N

H E C N A I R A V Y
Z S T A N D A R D B
L I M I T T A B L E
Z M E R O E H T X Y

INSTRUCTION/DELIVERY
As the sample size increases, the sampling distribution of the mean ( μ), can be
σ
approximated by a normal distribution with mean¿) and standard deviation ( ) . It means that if
√n
the sample sizen is randomly selected from a population with mean μ and varianceσ2, the
sampling distribution of the sample means will approach a normal distribution even when the
original population is not normally distributed, as long as the sample size n is sufficiently large.
Let us consider this example.
Illustrative Example: Consider a population consisting of the values 1, 2, 3, 4, 5, and 6.
Compute for the following:
population mean,
population variance and
population standard deviation.

Solution:
Σx
Get the mean by the formula μ=
n
where μ – population mean
Σ x −¿ sum of all the terms in x
N - sample size

Σx
μ= Substitute
N
1+ 2+ 3+4 +5+6
= Add each term of the sample divided by the sample size
6
21
= Simplify
6
μ = 3.5 population mean

To solve for the population variance, subtract each mean by the population mean and square the
result.
1-3.5 = -2.5 (2.5)2 = (2.5)(2.5) = 6.25
2-3.5 = -1.5 (-1.5)2 = (-1.5)(-1.5) = 2.25
3-3.5 = -0.5 (-0.5)2 = (-0.5)(-0.5)= 0.25
4-3.5 = 0.5 (0.5)2 = (0.5)(0.5) = 0.25
5-3.5 = 1.5 (1.5)2 = (1.5)(1.5) = 2.25
6-3.5 = 2.5 (2.5)2 = (2.5)(2.5) = 6.25
Σ (x -μ ¿2 = 17.5
Using the formula to solve the population variance

Σ( x−μ)2
σ2 = Substitute to the formula
N
17.5
=
21
σ2 = 0.833… population variance

To solve for the population standard deviation use the formula,


Σ( x−μ) 2
σ =√
N
17 . 5
=
= √ 0.833 ..

21

σ = 0.913 population standard deviation

MATH REAL TALK


The Central Limit theorem comes in whenever there is sufficient number of randomly
selected, independent samples (or observations), the means of those samples will follow
a normal distribution even if the population you’re sampling from does not.

AN ILLUSTRATION OF NORMAL DISTRIBUTION OF OBSERVATION

1 2 3 4 5 6
Mean

The illustration above is curve of normal distribution. Below, is an example of abnormal


distribution, it is very unlikely to happen that when there will be an equal chance of getting 1, 2, 3,
4, 5, and 6 in an observation.
Standard
Deviation
P(x
)

1 Standard
6 Deviation

1 2 3 4 5 6
X

The probability histogram of the population is shown above. Here n = 1. Since there are 6 possible
1
outcomes and each sample will have a chance of occurring once, then the probability is .
6
To further understand these concepts, tables are very useful. Taking them one at a time makes it
easier to see how each data reveal its value.
Observation x x-μ (x -μ ¿2 P (X=x)
1
1 1 -2.5 6.25
6
1
2 2 -1.5 2.25
6
1
3 3 -0.5 0.25
6
1
4 4 0.5 0.25
6
1
5 5 1.5 2.25
6
1
6 6 2.5 6.25
6
Σx=21 Σ (x-μ ¿2= 17.5 Σ P ( X =x )=1

Σx[ P ( x ) ]
The population mean μcan also be obtained using probabilities. Using the formula: μ=
N
where, μ− population mean
P(x) – probability that the sample will occur
Σx[ P ( x ) ]
μ=
N
1
= 21 ( ) Substitute and simplify
6
μ= 3.5
If you are going to look closely at the histogram, the shape of the curve as represented by the
graph is coming closer and approximately similar to the normal distribution curve compared to the
first histogram where the sample size is 1. Remember that the graph only considers the sample
size of 2.
1. What do you think will happen to the graph when the sample size is 3? 4? 5? 6?
2. What do you think is the mean of the sampling distribution of the means? Will it be different
from the population mean of 3.5? Or will it be the same?
3. If the sample size increases, what do you think will happen to the probability histogram of
the sampling distribution of the means?
4.

MATH REAL TALK


1. If the samples of size n, where n is sufficiently large, are drawn from any
population with a mean μ and standard deviation σ, then the sampling
distribution of the means approximates a normal distribution. A sample size of 30
is sufficient to approximate the normality of the distribution.
2. The sampling distribution of the sample of the means taken with replacement
from a population N with population mean and variance will approach a normal
distribution according to the Central Limit Theorem
3. As the sample size n increases, the sampling distribution of the means
approaches a normal distribution.
4. The mean of the sampling distribution of the means is equal to the mean of the
population.
5. The variance of the sampling distributions of the means is equal to the variance
of the population divided by the sample size n.

PRACTICE
Direction: Read and answer the following questions independently. Show your solution if
necessary. Write your answer in your activity/assessment notebook.
1. If the population mean is 12.45, what is the mean of the sampling distribution of its means?
2. If the mean of the sampling distribution is 24.29, what is the mean of the population?
3. If a population has a variance of 6.4, what is the variance of the sampling distribution of the
means if the sample size is 4 and all the possible samples are drawn with replacements?
4. If the population has a standard deviation of 9.4, what is the standard deviation of the sampling
distribution of its means? The sampling distribution was derived with sample size n=3, and all the
possible samples were drawn with replacements.
5. If the population standard deviation is 5.6, what is the population variance?

LESSON 5
Solving Problems Involving Sampling Distributions of the Sample Mean
INTRODUCTION
In the previous topics, you have learned how to use the normal distribution to gain
information about an individual data value obtained from population. In this lesson, you will use the
sampling distribution of the mean to obtain information about the sample mean.

A sampling distribution of sample means is a frequency distribution using the means


computed from all possible random samples of a specific size taken from a population. The
probability distribution of the sample means is also called the sampling distribution of the sample
means.
OBJECTIVES/COMPETENCIES – ESTIMATED TIME

Solves problems involving sampling 2 days


distributions of the sample mean
PRE-ASSESSMENT
Activity 1– PRE-ASSESSMENT
Direction: Read each statement carefully. Choose the corresponding letter that corresponds to
the best answer. Write your answer in your activity/assessment notebook. Label your work as Q3-
Lesson 5- Activity 1- Pre-Assessment.
For numbers 1 and 2, refer to the problem below. Use the Table of Areas under the Normal Curve
also known as z – Table for the probabilities.
The IQ of 300 students in a certain school in Quezon Province is approximately
normally distributed with μ= 100 and σ = 15.

1. What is the probability that a randomly selected student will have an IQ of 115 and
above?
A. 20.87% B. 15.87% C. 10.87% D. 5.87%
2. How many students have an IQ from 85 to 120?
A. 200 B. 210 C. 215 D. 225
For numbers 3 and 4, refer to the problem below.
3. If a single egg is selected, what is the probability that the cholesterol content will be more
than 220 milligrams?
A. 0.3707 B. 0.8523 C. 0.4562 D. 0.1258
4. If a sample of 25 eggs is selected, what is the probability that the mean of the sample will be
larger than 220 milligrams?
A. 0.1932 B. 0.2241 C. 0.0475 D. 0.1556
5. Two out of five adult smokers acquired the habit by age 14. If 400 smokers are randomly
selected, find the probability that 170 or more acquired the habit by age 14.
A. 0.8907 B. 0.166 C. 0.4156 D. 0.287

INSTRUCTION/DELIVERY
X−μ
z= is used when dealing with an individual data obtained from the population.
σ
X́−μ
z= σ is used when dealing with data about the sample means.
√n
Individual values are more variable than sample means.
The use of the two formulas in problem solving will be discussed here in the given problems.
The average time it takes a group of college students to complete a certain examination is 46.2
minutes. The standard deviation is 8 minutes. Assume that the variable is normally distributed.
(a) What is the probability that a randomly selected college student will complete the
examination in less 43 minutes?
To answer the problem, follow the steps:

Step 1. Identify the given information:


μ = 46.2
σ =8
X = 43
Step 2. Identify what is asked for:
P(X ˂ 43)
Step 3. Identify the formula to be used:
The problem is dealing with an individual data obtained from the population so the formula to be
X−μ
used is z = to standardize 43.
σ
Step 4. Solve the problem:
X−μ
z=
σ

43−46.2
=
8

= - 0.40

Find P(X¿ 43) by getting the area under the normal curve.
P(X˂ 43) = P(z ˂ -0.40)
= 0.5000 – 0.1554
= 0.3446
Therefore, the probability that a randomly selected college student will complete the examination
in less than 43 minutes is 0.3446 or 34.46%
(b) If 50 randomly selected college students take the examination, what is the probability that
the mean time it takes the group to complete the test will be less than 43 minutes?
Step 1: Identify the given information:
μ= 46.2
σ =8
X́ = 43
n= 50
Step 2: Identify what is asked.
P( X́ ˂ 43)
Step 3: Identify the formula to be used.
The problem is dealing with data about the sample means, so the formula to be used to
X́−μ
standardize 43 isz = σ
√n

Step 4: Solve the problem:


X́−μ
z= σ
√n
43−46.2
= 8
√ 50
= - 2.83

-2 -1 0 1 2

Find P( X́ ˂ 43) by getting the area under the normal curve.


P( X́ ˂ 43) = P(z ˂ -2.83)
= 0.5000 – 0.4977
= 0.0023
The probability that 50 randomly selected college students will complete the test in less
than 43 minutes is 0.0023 or 0.23%
In the two problems presented and solved, the probability of (a) is larger than (b) because
individual values are more variable than the sample means.
PRACTICE
Solve the following problems. Write your answer in your activity/assessment notebook.
1. The average number of milligrams (mg) of cholesterol in a cup of certain brand of ice cream is
660 mg
and the standard deviation is 35 mg. Assume the variable is normally distributed.
(a) If a cup of ice cream is selected, what is the probability that the cholesterol content will be
more than
670 mg?
Step 1:
_________________________________________________________________________
Step 2:
_________________________________________________________________________
Step 3:
_________________________________________________________________________
Step 4:
_________________________________________________________________________

CHAPTER IV: ESTIMATION OF PARAMETERS


LESSON 1
Illustrating t - distribution

INTRODUCTION
This module is about the t-distribution (Student’s t-distribution) and its properties. This will
illustrates the t-distribution and discuss its uses. After completing this module, the students are expected to
illustrate the t-distribution, enumerate its properties and state its similarities and/ or difference with the z-
distribution. It is assumed that the student already learned the topic about the normal distribution before
proceeding in this module.
OBJECTIVES/COMPETENCIES – ESTIMATED TIME

Illustrates point and interval estimates 6 days


Distinguishes between point and interval
estimations

PRE-ASSESSMENT
Activity 1– PRE-ASSESSMENT1
Direction: Answer the following first before you proceed with the module.Read each item carefully
and shade the letter of the correct answer.
1. When do we say that the sample size is sufficiently large?
A. When it is greater than or equal to 10.
B. When it is greater than or equal to 20.
C. When it is greater than or equal to 30.
D. When it is greater than or equal to 40.
2. In Student’s t distribution, if the sample size is 25, what is the degree of freedom?
A. 5 B. 24 C. 25 D. 26
3. What is the difference between the normal distribution and the t distribution?
A. The t distribution is centered at 0.
B. The t distribution is symmetric at the middle.
C. The t distribution has thicker tail.
D. The t distribution has higher peak.
4. When do we use t distribution instead of normal distribution?
A. When the sample size is less than 30.
B. When the sample size is greater than or equal to 30
C. If the population standard deviation is known
D. If the sample standard deviation is unknown
5. The labor department claims that the average starting salary of surveyors in Mindanao is
P24,000 per month. A sample of 15 surveyors has a mean of P23,220 and a standard deviation of
P400. In order to test the agency’s claim, what will you use.
A. t test B. z test C. p test D. f test

MOTIVATION
Activity 2–“ To z or not to z. That is the question”
Direction: In each illustration on estimation about the population, fill in the missing boxes with “to
z” if it requires you to use the z-distribution and “not to z” if it does not.Copy the format and write
your answer in your activity/assessment notebook.

1
_
m
a
e
h
t
s
I
z
i
l
p

S
E
Y
N
O
?
0
3
n
a
h
t
s
e
l
_
_
e

_
r
n
e
v
i
h
t
s
I
g
a _
Y
N
S
E
O
a
l
u
p
n
o
t
d
i
s
t
a
i
v
e
?
n
o
d
_
2
r
a
d
Process Questions:
Direction: Answer the questions that follows:Write your answer in your activity/assessment
notebook.
1. How did you find the activity?
2. Was it hard for you to fill in each box?
3. Based on the activity, when do we use the z-distribution? When don’t we use the z-distribution?
4. If the sample size is less than 30 and the population standard deviation is unknown, what do
you use instead of the z-distribution?

INSTRUCTION/DELIVERY
According to the Central Limit Theorem, the sampling distribution of a statistic (like a
sample mean) will follow a normal distribution, as long as the sample size is sufficiently large.
Therefore when we know the standard deviation of the population, we can compute a z-score and
use the normal distribution to evaluate probabilities with the sample mean.
But sample sizes are sometimes small, and often we do not know the standard deviation of
the population. When either of these problems occurs, statisticians rely on the distribution of the t-
statistic (also known as t-score). The distribution of the t-statistics is called t-distribution or
Student’s t-distribution that was created by William T. Gosset, an Irish brewery worker. Student’s t
distribution is very much like the standard normal distribution that it is centered at 0 and has the
same bell shape, but it has heavier tails than the standard normal distribution does.
The t distribution allows us to conduct statistical analyses on certain data sets that are not
appropriate for analysis using the normal distribution. Some of the properties of the t-distribution
are as follows:
1. The t-distribution is bell-shaped and symmetric about the mean
2. The t-distribution is a family of curves, each determined by the degree of freedom.
The degrees of freedom are the number of free choices left after a sample statistic is
calculated. It is equal to one less than the sample size. Df=n-1
3. The total area under a t-curve is equal to 1 (or 100%)
4. The mean, median and mode of the t-distribution are equal to zero.
5. As the degrees of freedom increase, the t-distribution approaches the normal distribution.

PRACTICE

Direction: Answer the following questions. Write your answer in your activity/assessment
notebook.
1. If the sample size is 25, what is the degree of freedom?
2. Both the normal distribution and t distribution have their centers at 0, true or false?
3. Like the normal distribution, the t distribution is asymptotic to which axis, x or y?
4. What is the total area under the t curve?
5. When do we say that the sample size is sufficiently large?

You might also like